Vallankumouksen sirpaleita

2.12.2018 klo 12.24, kirjoittaja
Kategoriat: Kosmokseen kirjoitettua

Matemaattisten aineiden opettajien liiton lehdessä Dimensio 4/2018 on artikkelini Vallankumouksen sirpaleita: huomioita fysiikan opetuksesta. Juttua ei ole nettiversiossa, kiinnostuneiden täytyy haalia käsiinsä paperia. Kirjoitus alkaa seuraavasti:

”Fysiikka on vallankumouksellista. Vuosisata sitten kvanttifysiikka ja suhteellisuusteoria mullistivat ymmärryksemme todellisuudesta. Suhteellisuusteoria muutti käsityksen ajasta ja avaruudesta. Tämä avasi ovet maailmankaikkeuden kehityksen hahmottamiseen ja ihmisten näkemiseen osana maailmankaikkeuden historiaa. Kvanttifysiikka paljasti aineen luonteesta asioita, jotka olivat kuvitelmia ihmeellisempiä. Kvanttifysiikka on myös osoittautunut kaikkien aikojen hedelmällisimmäksi tieteelliseksi teoriaksi: se muodostaa elektroniikan ja nykykemian –toisin sanoen lähes kaiken nykyteknologian– pohjan.

Koulufysiikan opetussuunnitelmassa näitä asioita hädin tuskin mainitaan.”

12 kommenttia “Vallankumouksen sirpaleita”

  1. Cargo sanoo:

    Itse taas en voi kuin ihmetellä, että miksei pseudo-Riemannin monistojen kovariantti derivaattaa tai Hilbertin avaruuden rajoittamattomien operaattoteiden spektraaliteoriaa mainita koulumatematiikan opetussuunnitelmassa.

    1. Syksy Räsänen sanoo:

      Jos haluat lukea, mistä itse asiassa on kysymys, niin kirjoitin Ylelle joku aika sitten samasta aiheesta suppeamman jutun:

      https://yle.fi/aihe/artikkeli/2018/04/13/opetetaanko-koulussa-fysiikkaa-vaarin-syksy-rasanen-opetuksen-sisalto-on

      1. Cargo sanoo:

        Kiitosta vain linkistä, ja itse voin lukea tuon Dimension paikallisen yliopiston kirjastossa. Tosin koko läpyskä on nykyään niin täynnä pelkkää digihöttöö ja temppuoppimista, ettei paljoa jaksa vilkaista.

        Mutta kun osaaminen tulee tulevaisuudessa edelleen laskemaan, niin ehkä Dimensionkin sivut vielä täyttyvät konservatiivisista vaatimuksista tasoryhmien ja opettajajohtoisen kynä-paperi-opiskelun puolesta.

      2. Cargo sanoo:

        Olisiko mahdollista kehittää lukion fysiikkaan valinnaista kurssia, esim. Physics X – á la Feynman? Nykyinen päättömästi digiloikkaava opetusministeri tuntuu innostuvan aivan kaikesta, jos siinä on jotain uutta. Fysiikasta kiinnostuneet oppilaat ovat toki jo nyt netin kautta autodidaktisia, mutta laajempi oppilasjoukko voitaisiin houkutella tieteen maailmaan tuollaisen valinnaisen kurssin kautta, etenkin kun ottaa huomioon esim. poikien luontaisen laumakäyttäytymisen.

        Tuollaisella Fysiikka X -kurssilla sitten pohditaan opettajan johdolla luonnonilmiöitä ja niistä seuraavaa uutta fysiikkaa minimaalisin matemaattisin apuvälinein. Jos esim. oletetaan Maxwellin yhtälöt tunnetuiksi, niin tarkastelemalla kahta fysikaalisesti erillistä tilannetta, joissa magneetti liikkuu johtimen ollessa levossa, tai johtimen liikkuessa magneetin ollessa levossa, päädytää suhteellisuusperiaatteeseen että havainnot saataisiin selitettyä. Sitten voitaisiinkin tarkastella graafisesti aika-avaruus-koordinaatistoa ja suhteellisuusperiaatteen edellyttämää Lorentz-muunnosta, josta päädytäänkin aikadilataatioon ja vaikka relativistiseen liikemäärään. Kvanttimekaniikkaa voitaisiin selittää fotonien ja elektronien samanlaisella diffraktiolla, mistä seuraa selitys valon ja ainehiukkasten liikkeelle kaikkien eri ”vaiheisten” polkujen summana. Visuaalisesti havainnollistettu Fourier-muunnos ja kokeellinen epätarkkuusperiaate selittää hiukkasten aaltomaisuuden sekä atomien spektrin seisovina aaltoina potentiaalikuopissa yms. yms. Eräs ohjenuora voisi olla se, että kaikki fysiikan teoriat pyritään ”johtamaan” ensimmäisten periaatteiden sekä yksinkertaistetun matematiikan avulla.

  2. Jos ei muuta, niin ainakin sen voisi kaikillekin opettaa että ilman kvanttifysiikkaa normaalia ainetta ei voisi olla olemassa, koska elektronit etsiytyisivät atomiytimiin ja jäisivät sinne. (Formulointia pitäisi toki miettiä huolellisemmin.)

  3. Tapsa sanoo:

    Syksy, nyt olet ihan asian ytimessä! Kannatan!

    On aivan käsittämätöntä, ettei suurimmalla osalla (näin rohkenen oman kokemukseni perusteella väittää) muuten fiksuista ja koulutetuista ihmisistä ole oikein mitään käsitystä näistä asioista, jotka on kuitenkin kehitetty jo 100 vuotta sitten.

    Puhuin kerran erään ict-ammattilaisen (!) kanssa avaruusmatkoista ja viittasin ohimennen ns. kaksosparadoksiin. Kaverini ei tuntenut termiä, vaan pyysi selittämään asian – ja sen lopuksi tokaisi: valehtelet.

    1. Vertailun vuoksi, kaikenlaista on tullut vastaan. Esimerkiksi väittelin kerran pitkään erään ulkomaisen avaruusplasmafyysikon kanssa eräästä matemaattisesta jutusta. En antanut asiassa periksi, koska hänen väitteensä oli että kompleksiluvun neliö on reaaliluku. Olen myös tavannut useamman kuin yhden satelliitteja työkseen suunnittelevan insinöörin, jolle mekaniikan perusasiat kuten pyörismäärän säilymislaki ja Newtonin kolmas laki tuntuvat olevan uusia asioita.

      1. Cargo sanoo:

        Itse olen ollut seuraamassa väitöstilaisuutta, jossa vastaväittäjä kysyi, että ”mistä tämä tulos XX on tullut?” Kyseessä oli siis simppeli moniulotteisen differentiaaliyhtälön ratkaisu, jossa eksponentti oli muotoa (vakio)*|x|^2. Tuleva tohtori oli vain kopioinut tuloksen jostain eikä millään muistanut, että neliö tarkoittaa reaalisten vektoreiden pistetuloa! Piruparka meni aivan jäihin ja vasta kustoksen puuttuminen tilanteeseen palautti väitöskeskustelun takaisin raiteilleen. Toisaalta myös Werner Heisenberg mokasi väitöstilaisuutensa, kun Wilhelm Wien alkoi kuulustelemaan kokeellisesta fysiikasta ja sai teoreettisen väittelijän täysin hämilleen.

      2. Eusa sanoo:

        Jospa hän tarkoitti/muisteli, että kompleksiluvun normin neliö eli luvun ja sen kompleksikonjugaatin tulo on aina reaaliluku?

  4. Erkki Kolehmainen sanoo:

    Upetuksen kehittäminen on jalo pyrkimys, koska opetus jää helposti lapsipuolen asemaan painopisteen ollessa tutkimuksessa. En kuitenkaan kantaisi asiasta liian suurta huolta, sillä asioista kiinnostunut löytää netistä todella hyvää materiaalia, joka voi poiketa myös yleisestä paradigmasta.

    Kannattaa muistaa Stanislaw Lemin aforismi;

    ”Monta asiaa olisin oppinut, ellei niitä olisi minulle opetettu.”

    1. Syksy Räsänen sanoo:

      Nettivideot eivät korvaa kunnollista kouluopetusta.

  5. Lentotaidoton sanoo:

    Itse olen sitä ikäluokkaa, jolle oppikoulun (siis tällaisen muinaissysteemin) lukiossa ei tietenkään puolella sanalla puhuttu suhteellisusteoriasta saati sitten että sanaa kvantti olisi edes kukaan koskaan kuullutkaan. Einstein vain nimenä oli hädin tuskin tuttu (muista ei ollut aavistustakaan). No eipä yhteiskunnan tekniikkakaan ollut muutenkaan kummoista, höyryjunat puuskuttivat ja ensimmäiset televisiot (musta-valkoiset) ilmestyivät äveriäimpien kotiin (ja Ifalla ja piikkinokkaMossella ajettiin).

    Kirjoitin kuitenkin ns pitkän matikan. Jota senkoomin en ole tarvinnut kuin yksinkertaisten ensimmäisen asteen yhtälöiden ratkaisuissa. Ainoa yksityiskohta muistissa kouluajalta oli kun ns prelluissa kysyttiin derivaatta sini x:n ratkaisua. Olen saanut kaupallisen koulutuksen sekä senmukaisen ”karriäärin”. Tie siitä eteenpäin on ollut (tuskaisen) pitkää itseopiskelua. Kukaan eikä mikään pakottanut. Tiedon jano yksinkertaisesti yllytti ottamaan asioista selvää. Nyt eläkeukkona luen joka päivä sekä suhteellisuusteoriaa että kvanttifysiikaa ja etenkin kosmologiaa (nämähän kaikkihan nivoutuvat läheisesti toisiinsa) ja tämä enimmäkseen englanniksi. Suuri rajoitteeni edelleen on matematiikka.

    Maailma on nimenomaan tekniikan osalta kehittynyt noista ajoista käsittämättömän rajusti. Monasti ajattelen että mitähän minustakin olisi tullut jos olisi koulussa edes välttävän alustavat tiedot annettu suhteellisuudesta/kvanteista. Olisiko ura heittänyt toisaalle? Periaatteessa nyt tilanne on totaalisti toinen. Vaikka koulussa edelleen vaietaan noista asioista, niin netin/television (ja Räsästen yms) ansiosta jotkut saavat kuitenkin edes sen alustavan kipinän (tietysti Räsäsen mukaisesti tämä ei vastaa varsinaista opiskelua).

Vastaa

Sähköpostiosoitettasi ei julkaista. Pakolliset kentät on merkitty *

Sormustimen verran

28.11.2018 klo 13.55, kirjoittaja
Kategoriat: Kosmokseen kirjoitettua , Kosmologia

Maailmankaikkeuden koosta kysytään usein, joten avaan tässä asiaa hieman.

Keskeinen kysymys on se, onko maailmankaikkeus äärellinen vai ääretön. Mukaillen Arthur C. Clarken kommenttia kysymyksestä, onko Maapallon ulkopuolella elämää vai ei: kumpikin vaihtoehto on ihmeellinen. Ja tässäkään tapauksessa emme tiedä kumpi pitää paikkansa.

Maailmankaikkeuden ikä on äärellinen ja valo kulkee äärellisellä nopeudella, joten näemme vain äärellisen etäisyyden päähän. Tämä etäisyys rajaa kosmisen horisontin, jonka tuonpuoleisesta meillä ei ole mitään havaintoa. Maailmankaikkeus on 14 miljardia vuotta vanha, joten jos maailmankaikkeus ei laajenisi, horisontti olisi 14 miljardin valovuoden päässä.

Avaruuden laajeneminen muuttaa horisontin etäisyyttä kahdella tavalla. Ensinnäkin valo kulkee tietyssä ajassa lyhyemmän matkan: valoa hidastaa se, että sen reitille syntyy koko ajan lisää tilaa, joka pitää tarpoa. Toisaalta etäisyys valonsäteen alku- ja loppupisteen välillä kasvaa. Tämä seikka kasvattaa myös sitä osaa matkasta, jonka valo on jo kulkenut: koko väli alusta loppuun venyy kuin kumimatto.

Jälkimmäinen vaikutus on isompi: näemme nykyään 50 miljardin valovuoden päähän. Tämä ei tarkoita, että näkisimme maailmankaikkeuden laajenemisen takia kohteita, joita emme muuten olisi nähneet. Päin vastoin, jos maailmankaikkeus ei laajenisi, meille olisi ehtinyt tulla valoa useammasta galaksista kuin nyt. Mutta ne alueet, jotka olemme nähneet, ovat työntyneet kauemmas avaruuden laajetessa.

Jos maailmankaikkeus on äärellinen, niin kulkemalle tarpeeksi kauan samaan suuntaan palaa lopulta alkupisteeseen, kuin pallon tai donitsin pinnalla. Tämä pätee myös valonsäteisiin. Mitään merkkejä tällaisesta ei ole nähty, joten maailmankaikkeuden koko on isompi kuin näkemämme 50 miljardia valovuotta.

Meillä ei ole mitään varmaa tietoa siitä, millainen maailmankaikkeus on horisontin takana, hyvin perusteltuja arveluita kylläkin. Näkemämme osa maailmankaikkeutta on tilastollisesti samanlainen kaikkialla. Jos tarkastelee kuutiota, jonka sivun pituus on vähintään 500 miljoonaa valovuotta, niin sen keskimääräiset ominaisuudet ovat samanlaiset olipa kuutio missä tahansa: galakseja ja muuta ainetta on joka kuutiossa saman verran ja ne muodostavat samanlaisia kokonaisuuksia.

Kosminen inflaatio selittää miksi maailmankaikkeus on samanlainen joka puolella. Sen mukaan kiihtyvä laajeneminen ensimmäisen sekunnin murto-osan aikana paisutti nyt näkemämme alueen valtaviin mittoihin. Kiihtyvä laajeneminen pyyhki pois kaikki inflaatiota edeltäneet epätasaisuudet, jäljelle jäi vain inflaation kvanttivärähtelyissä syntyvät rakenteen siemenet. Ne ovat samanlaisia kaikkialla, joten niin ovat niistä myöhemmin kasvavat galaksit ja muut rakenteet.

Nykyään näkemämme maailmankaikkeuden osan säde oli inflaation alussa alle 10^(-23) metriä, eli sata miljoonaa kertaa protonia pienempi. Kun aine syntyi inflaation lopussa, alueen säde oli noin senttimetrin verran, koko tunnettu maailmankaikkeus mahtui sormustimeen. Kun maailmankaikkeuden kvarkit sitoutuivat protoneiksi ja neutroneiksi 30 mikrosekunnin aikaan, sen koko oli 20 valopäivää. Kun protonit ja neutronit muodostivat atomiytimiä neljän minuutin aikaan, koko oli 20 valovuotta. Kun atomiytimet yhtyivät elektroneihin muodostaen atomeita 380 000 vuoden iässä, nyt näkemämme alueen säde oli 50 miljoonaa valovuotta.

Emme tiedä kauanko inflaatio kesti tai paljonko se kaikkiaan venytti avaruutta. Emme siis osaa sanoa kuinka pitkälle maailmankaikkeudessa jatkuu inflaation tasoittama osuus ja samanlaiset rakenteet kuin ne mitä olemme nähneet. Olisi kuitenkin aikamoinen sattuma, jos näkisimme juuri nyt sen alueen reunalle, jonka inflaatio suurensi. Luultavasti maailmankaikkeus näyttää samanlaiselta vielä satojatuhansia kertoja isommalla mittakaavalla: vaikka matkaisimme miljoonien miljardien valovuosien etäisyyksille, siellä olisi samanlaista kuin täällä.

Se, millainen maailmankaikkeuden rakenne on kaikkein suurimmassa mittakaavassa –vai jatkuuko avaruus äärettömiin– liittyy siihen, millainen maailmankaikkeus oli ennen inflaatiota. Siitä meillä ei ole kunnollista teoreettista käsitystä eikä minkäänlaisia havaintoja.

40 kommenttia “Sormustimen verran”

  1. Cargo sanoo:

    Olisiko mitenkään mahdollista, että inflaation aikana referenssikellon viisarit pyörivät kuin hedelmäpeli ja sen takia avaruus laajeni nopeammin kuin tämänhetkinen valonnopeus? Jos siis aika määritellään entropian muutoksena, niin totta kai alkupamauksen aikana maailmankaikkeuden entropia lisääntyi vauhdilla, mutta tänäpäivänä entropia kasvaa suunnilleen samaa tahtiin kuin eilen.

    Ja jos aineen jakauma on kaikkialla samanlainen by default, niin itse avaruuden tulee laajeta, että entropia voisi kasvaa? Maailmankaikkeus voi siis hyvinkin olla ääretön systeemi, joka toteuttaa universaalia entropiaperiaatetta?

    No taas kerran sorruin nojatuolifysisointiin, mutta eiköhän Eusa tule jakamaan syntiä omalla aivokohinallaan 😉

    1. Syksy Räsänen sanoo:

      Ei.

      Joskus käytetty ilmaisu ”laajenee valoa nopeammin” on epämääräinen ja sitä pitäisi välttää. Se viittaa siihen, että jos avaruuden laajeneminen kiihtyy, niin kaukaiset havaitsijat eivät voi enää lähettää toisilleen valonsätietä, koska heidän väliinsä syntyy tilaa niin nopeasti.

      Aikaa ei määritellä entropian muutoksena. Inflaation aikana havaittavan osan maailmankaikkeutta entropia laskee nopeasti.

      1. Cargo sanoo:

        OK. Mutta miten se aika fysiikassa sitten määritellään? Luulisi, että saman systeemin kahta eri entropiatilaa kuvataan ajan kautta, ja esim. vedyn perustilalla aika on pysähtynyt, koska elektroni ei voi liikkua eikä toisaalta olla paikoillaankaan (jos siis oletetaan muuten tyhjä maailmankaikkeus).

        Voiko sitä inflaatiota ja koko maailmankaikkeuden systeemiä tarkastella niin yleisellä tasolla, että se entropia todellakin kasvaa? Eli jos otetaan kaikki energiamuodot ja vuorovaikutukset huomioon? Ja onko kukaan edes yrittänyt selittää avaruuden laajenemista siten, että se mahdollistaa suljetun systeemin entropian kasvun?

        1. Syksy Räsänen sanoo:

          Yleisessä suhteellisuusteoriassa aika on yksi osa aika-avaruudetta, eikä sen olemassaololla tai eteenpäin kulkemisella ole mitään tekemistä entropian kanssa.

          Mutta sitä, miksi yleisessä suhteellisuusteoriassa on yksi ulottuvuus joka on erilainen kuin muut, ei tiedetä. Kuten ei myöskään sitä, miksi signaalit kulkevat ajassa vain yhteen suuntaan.

  2. JaniK sanoo:

    Kiitokset mielenkiintoisesta blogista, tätä on hauska lukea!

    Kaikkialla (populäärikirjallisuudessa) aina sanotaan, että on väärin ajatella avaruuden laajenevan johonkin, mutta mikä todistaa, että näin on? Tai miksei voida ajatella avaruuden olevan ääretön ja siellä sattumalta ”klöntti” ainetta lähti laajenemaan?

    1. Syksy Räsänen sanoo:

      Kiitos, mukava kuulla.

      Maailmankaikkeuden laajenemista on selitetty täällä: https://www.tiede.fi/blogit/maailmankaikkeutta_etsimassa/rajaton_kasvu

  3. Erkki Tietäväinen sanoo:

    Jatkan tyhmien kysymysteni sarjaa: Näemme nykyään kohteen, josta lähteneellä valolla on kulunut yli 13 miljardia vuotta saavuttaa meidät. Olemme siis tänään noin pitkän matkan päässä paikasta, jossa kohde sijaitsi yli 13 miljardia vuotta sitten. Nythän se sijaitsee ties missä. Miten on mahdollista, että olemme niin kaukana toisistamme? Ymmärtäisin asian, jos inflaatio olisi loitontanut avaruuden osia toisistaan huikeasti valoa nopeammin, mutta niinhän ei ole tapahtunut. Jos kerran nyt näkemämme maailmankaikkeus oli alkuräjähdystä seuranneen inflaation alussa sata miljoonaa kertaa protonia pienempi, selitykseksi ei taida käydä se, että ”maailmankaikkeus ei syntynyt yhdessä pisteessä, vaan joka paikassa samaan aikaan”.

    1. Syksy Räsänen sanoo:

      ”Miten on mahdollista, että olemme niin kaukana toisistamme?”

      Koska maailmankaikkeus on laajentunut.

      1. Erkki Tietäväinen sanoo:

        Toki minäkin ymmärrän, että maailmankaikkeus on laajentunut. Se ei kuitenkaan ole vastaus kysymykseeni. Yritän tehdä kysymykseni uudella tavalla:

        Kun maailmankaikkeudella on ollut aikaa laajeta (reippaasti valoa hitaammin) vain 13,8 miljardia vuotta, niin en ymmärrä, miten voimme sijaita jo yli 13 miljardin päässä jostain kohteesta puhumattakaan, että näkemämme maailmankaikkeuden koko on peräti 50 miljardia valovuotta. Millä nopeudella laajeneminen siis on tapahtunut?

        1. Syksy Räsänen sanoo:

          Maailmankaikkeuden laajenemisessa ei ole kyse siitä, että kappaleet liikkuisivat poispäin toisistaan. Sen sijaan avaruuden tilavuus kasvaa. (Valonnopeudella ei siis ole mitään tekemistä asian kanssa.)

          Laajenemisnopeus muuttuu ajan myötä. Tällä hetkellä se on noin 70 km/s/Mpc, missä Mpc on noin 3 miljoonaa valovuotta. Mitä kauempana kohde on, sitä nopeammin se näyttää etääntyvän meistä, koska sitä enemmän väliimme tulee uutta tilaa.

          Inflaation jälkeen laajenemisnopeus on ollut kaikkina aikoina suuruusluokkaa 1/t, missä t on maailmankaikkeuden ikä.

          1. Erkki Tietäväinen sanoo:

            Sanot näin: ”Maailmankaikkeuden laajenemisessa ei ole kyse siitä, että kappaleet liikkuisivat poispäin toisistaan. Sen sijaan avaruuden tilavuus kasvaa.”

            Tarkoitatko siis, että varhaisin toistaiseksi näkemämme kohde yli 13 miljardin valovuoden päässä on ollut koko ajan yhtä kaukana meistä? Avaruusko vain on kasvattanut tilavuuttaan, mutta kohteen ja maapallon välinen etäisyys on pysynyt samana? Eihän se nyt näin voi olla, kun kertomasi mukaan kaikki alkoi sata miljoonaa kertaa protonia pienemmästä tilasta.

          2. ”Maailmankaikkeuden laajenemisessa ei ole kyse siitä, että kappaleet liikkuisivat poispäin toisistaan.” Hmm. Sanoisin että totta kai kaukaiset galaksit liikkuvat meistä poispäin: jos sellaisen kanssa vaihtaa radioviestejä, viestien aikaväli kasvaa. Tilan syntymisellä väliin tarkoitetaan kai sitä että kumpikin etääntyjä pysyy levossa universaalin lepokoordinaatiston eli mikroaaltotaustan suhteen. Mutta sen voi kai ajatella johtuvan homogeenisuudesta ja isotrooppisuudesta.

          3. Syksy Räsänen sanoo:

            Maailmankaikkeuden laajenemista ei voi ymmärtää havaitsijoiden liikkeenä pois toisistaan, eikä homogeenisuudella ja isotrooppisuudella ole mitään tekemistä asian kanssa.

          4. ”Maailmankaikkeuden laajenemista ei voi ymmärtää havaitsijoiden liikkeenä pois toisistaan.” Jos näin on, missä kulkee raja paikallisen ja globaalin välillä? Pitääkö esimerkiksi paikallisen galaksiryhmän galakseja ajatella normaalisti liikkuvina, mutta jostain isommasta pituusskaalasta lähtien pitääkin alkaa ajatella että väliin syntyy sen sijaan uutta avaruutta?

          5. Syksy Räsänen sanoo:

            Ainoastaan neliulotteinen aika-avaruus on määritelty yksikäsitteisesti, sen voi siivuttaa avaruudeksi ja ajaksi eri tavoilla.

            Kun on valittu tietty siivutus, paikallisesti yhdessä pisteessä havaitsija voi liikkua avaruuden (tai muiden havaitsijoiden) suhteen tai olla liikkumatta. Sen sijaan kysymys siitä, liikkuuko havaitsija jonkun sellaisen havaitsijan suhteen, joka ei ole samassa pisteessä ei ole mielekäs, koska ei ole yksikäsitteistä tapaa verrata eri pisteissä olevia nopeuksia.

            Jos valitsee joukon havaitsijoita, avaruuden voi määritellä siten, että he ovat aina paikallaan. Tällöin heidän suhteellisten etäisyyksiensä ja paikkojensa muutos selittyy avaruuden laajenemisella, muodon muuttumisella (kutistumisella ja venymisellä eri suuntiin) ja pyörimisellä.

          6. Ymmärrän vastauksesi muut osat, mutta jäin miettimään miksei eri pisteiden nopeuksia voisi verrata. Nelinopeus on maailmanviivan tangentti, ja nelinopeusvektoreita voi yrittää verrata yhdensuuntaissiirtämällä ne samaan pisteeseen. Siirto ei riipu tiestä kun avaruus on laakea. Olen ymmärtänyt että havaintojen mukaan avaruus on laakea.

          7. Syksy Räsänen sanoo:

            Siirto ei riipu tiestä kun aika-avaruus on laakea.

            Maailmankaikkeudessa aika-avaruus ei ole laakea: maailmankaikkeuden laajeneminen on tämän ilmentymä.

            Avaruus (ei aika-avaruus) on laakea keskimäärin, ei sekään paikallisesti.

          8. Aivan, niinhän se on. Kiitos vastauksista.

          9. eräs Esa vaan sanoo:

            Olen näissä(kin) tähtitieteen asioissa amatööri ja alkutaipaleen harrastelija, ja hiukan hävettää näin hölmö kysymykseni, liekö jo vastattu monia monituisia kertoja samanlasiin, paremmin muotoiltuihin kysymyksiin. Mutta kysyn silti.
            Koska aika on suhteellista, niin millä kellolla on avaruuden ikä ja muut ajallisesti määritellyt välimatkat mitattu?

          10. Syksy Räsänen sanoo:

            Sellaisen havaitsijan kellolla, joka liikkuu aineen mukana. (Näkyvän aineen ja pimeän aineen välinen nopeusero on niin pieni, että ei juuri ole väliä, kumman valitsee.)

  4. Lentotaidoton sanoo:

    Räsänen: ”Kuten ei myöskään sitä, miksi signaalit kulkevat ajassa vain yhteen suuntaan”.

    Eikö tämä ole (vakava) probleema? Fysiikan lait itsessään eivät tee eroa tulevaisuuden ja menneisyyden välillä, ajan suunnalle ei esitetä objektiivista fysikaalista perustaa. Tiedämme kuitenkin että heikko voima rikkoo CP-symmetriaa, mutta että yhdistetty CPT-symmetria (eli CPT–invarianssi) on rikkoutumaton (esim K- ja B-mesonien kohdalla) mutta että esim T-symmetria yksinään ei ole invariantti. Ja että CP-symmetrian on rikkouduttava, jotta CPT-symmetria ei rikkoutuisi. Eli nämä hiukkaset tunnistavat menneen ja tulevan (eli ajan ”nuolen”)?

    1. Syksy Räsänen sanoo:

      On. Ei tiedetä, miksi ajalla on määrätty suunta. Klassisessa sähkömagnetismissa ja yleisessä suhteellisuusteoriassa on yhtä lailla ajassa taaksepäin meneviä kuin ajassa eteenpäin kulkevia signaaleja, mutta jostain syystä maailmankaikkeudessa on ainoastaan jälkimmäisiä.

      1. Eusa sanoo:

        Voihan vapaata putoamista pitää retrokausaalisuutena; kappale putoaa siten kuin putoaa, jotta menneisyydessä voisi olla olemassa harvaa kaasua, josta tihentynyt alue ja jyrkentynyt kaarevuus. Jotta elliptisten kiertoratojen perusperihelikiertymä, vuorovaikutuksena mallinnettavien staattista gravitaatiokenttää ylläpitävien signaalien on kuljettava tulevaisuudesta menneisyyteen.

        Tätä ei toki selitetä näin fysiikassa, mutta voitaisiin selittää, jos esim. olisimme eri havaintotarkkuustilanteessa. Ainahan olemme jossain havaintotarkkuudessa kiinni – on havaitsematonta ilmiötä, joihin päästään käsiksi vasta kehittymisen myötä…

        1. Syksy Räsänen sanoo:

          Ei, kausaalisuuden suuntaa ei voi kääntää tuolla tavalla.

          On havaittu vain ajassa eteenpäin kulkevia signaaleja, ei koskaan ajassa taaksepäin kulkevia. Tämän takia menneisyys vaikuttaa tulevaisuuteen, mutta ei toisin päin.

          Tämä riittäköön tästä.

    2. Cargo sanoo:

      Voisiko ajan yksisuuntaisuutta perustella entropian ohella myös kvanttimekaniikalla? Kun hiukkasta havainnoidaan, niin aaltofuntio romahtaa ja uusi aaltofunktio muodostuu ajanhetkellä t=0 ja lähtee leviämään, eikä tällöin ole mitään mahdollisuutta saada tietoa menneisyydestä. Ottamalla lisäksi huomioon, että hiukkasta havainnoi aina ”jokin”, sillä maailmankaikkeus ei ole tyhjä, niin jokainen ajanhetki on tavallaan uusi t=0 ja näin ollen ajalla voi olla vain yksi suunta ja se on eteenpäin sarjana diskreettejä tilannekuvia. Ehkäpä yleinen suhteellisuusteoria on niin arkaainen viritelmä, ettei siihen pysty sovittamaan epädeterministisen entropian/informaation aiheuttamia dynaamisia muutoksia.

      Markiisi de Laplace pitää nyt lopullisesti heittää niska-perse-otteella ulos häiritsemästä tieteen kehitystä.

      1. Syksy Räsänen sanoo:

        Voi olla, että kvanttimekaniikalla on tekemistä asian kanssa. Tämän selvittämiseen tarvittaisiin ymmärrys kvanttimekaniikan mittausongelmasta ja/tai kvanttigravitaatiosta, mitä kumpaakaan meillä ei ole.

        Olen kirjoittanut täällä siitä, mitä tiedämme ajasta:

        https://www.ursa.fi/blogi/kosmokseen-kirjoitettua/kaksi-tarinaa-ajasta/

        https://www.tiede.fi/blogit/maailmankaikkeutta_etsimassa/ajan_kanssa

        https://www.tiede.fi/blogit/maailmankaikkeutta_etsimassa/ajankayton_hallinta

  5. pekkap0 sanoo:

    Onko mahdollista, että emme ole vielä keksineet vääntää universumia kuvaavia yhtälöitä sellaiseen muotoon, jossa tunnetut dimensiot xyzt korvautuvat uusilla ja jotka selittävät meille nykyään tuntemattomia ilmiöitä paremmin, vaikka eivät olekaan etäisyyksiin ja aikaan nähden yhtä käytännönläheisiä dimensioita?

    1. Syksy Räsänen sanoo:

      Yleisen suhteellisuusteorian mukainen neliulotteinen aika-avaruus kuvaa havaintojamme erinomaisesti.

      On tutkittu ideaa, jonka mukaan aika-avaruus on approksimaatio jostain perustavanlaatuisemmasta, mutta toistaiseksi asiasta ei tiedetä mitään varmaa, ks.

      https://www.ursa.fi/blogi/kosmokseen-kirjoitettua/kaikki-tai-ei-mitaan/

      https://www.ursa.fi/blogi/kosmokseen-kirjoitettua/aika-avaruuden-atomit/

  6. Timo K sanoo:

    Aiheeseen liittyen eräs mieltäni askarruttava kysymys: kosminen taustasäteily syntyi kun maailmankaikkeus oli 380.000 vuoden ikäinen.

    Eli yli 13 miljardia vuotta sitten.

    Miten pystymme edelleen havaitsemaan sitä, luulisi nimittäin että tuossa ajassa fotonit olisivat ehtineet joko suihkimaan ohitsemme, tai osumaan johonkin kohteeseen. Eli ei pitäisi olla enää mitään havaittavaa taustasäteilyä.

    Mutta on, miten se on mahdollista?

    1. Syksy Räsänen sanoo:

      Kosminen mikroaaltotausta syntyi kaikkialla. Näemme siis nyt kosmisen mikroaaltotaustasäteilyn fotoneita, jotka lähtivät liikkeelle paikasta, joka on meistä nyut 50 miljardin valovuoden päässä. Miljardi vuotta sitten tästä pyyhkivät ohi fotonit, jotka olivat lähteneet liikkeelle hieman lähempää meitä, ja niin edelleen.

      Kosminen mikroaaltotausta siis muuttuu koko ajan, koska näemme eri hetkinä eri fotoneita, mutta muutosnopeus on hyvin pieni, koska lähellä toisiaan liikkeelle lähteneet (ja melkein saman ajan kulkeneet) fotonit ovat hyvin samanlaisia.

  7. Juha sanoo:

    Tuohon inflaatioon ja aineen kasautumiseen liittyen; Millaisia ilmiöitä avaruuden kaarevuuden muutos aiheuttaa, vai aiheuttaako minkäälaisia? Esim. jos mustan-aukon aine jostain kumman syystä äkillisesti levittäytyisi laajalle alueelle tasaisesti, olisiko sillä mitään yhteyttä ilmiöiden kanssa, jotka liittyvät avaruuden laajenemiseen (punasirrtymä tms.)?

    1. Syksy Räsänen sanoo:

      Avaruuden laajeneminen on aika-avaruuden kaarevuuden ilmentymä, ei avaruuden kaarevuuden.

      Kaikki gravitaation ilmiöt (avaruuden laajeneminen, kappaleiden välinen näennäinen vetovoima, valon taipuminen jne.) ovat aika-avaruuden kaarevuuden ilmentymiä. Aineen jakauma määrää aika-avaruuden kaarevuuden.

  8. Mika sanoo:

    Kiitos taas mielenkiintoisesta blogista, tämä on yksi parhaita suomenkielisiä tiedeblogeja.

    Jatkan hiukan samasta aiheesta aiemman kommentoijan kanssa, ja kysyisin, että kun kirjoitat ”Kun aine syntyi inflaation lopussa, alueen säde oli noin senttimetrin verran, koko tunnettu maailmankaikkeus mahtui sormustimeen.” ja taas toisaalta 30 mikrosekunnin aikaan koko oli jo 20 valopäivää ja 380 000 vuoden kohdalla 50 miljoonaa valovuotta, niin onko alkuvaiheessa inflaation jälkeen laajeneminen ollut vielä nopeampaa ja pikkuhiljaa hidastunut, kunnes alkoi taas kiihtyä myöhemmin?

    Mikäli näin on, niin mikä sai aikaan tuon nopean laajenemisen inflaation jälkeen? Jäikö inflaatiosta jonkinlaista laajenemisenergiaa jota gravitaatio pikkuhiljaa hidasti?

    1. Syksy Räsänen sanoo:

      Kiitos, mukava kuulla.

      Yltä siteerattua:

      ”Laajenemisnopeus muuttuu ajan myötä. Tällä hetkellä se on noin 70 km/s/Mpc, missä Mpc on noin 3 miljoonaa valovuotta. Mitä kauempana kohde on, sitä nopeammin se näyttää etääntyvän meistä, koska sitä enemmän väliimme tulee uutta tilaa.

      Inflaation jälkeen laajenemisnopeus on ollut kaikkina aikoina suuruusluokkaa 1/t, missä t on maailmankaikkeuden ikä.”

      Viimeisen muutaman miljardin vuoden kuluttua laajeneminen on tosiaan taas kiihtynyt.

      Laajeneminen on inflaation jälkeen nopeaa, koska se oli inflaation aikaan nopeaa. Inflaation aikaan se oli nopeaa, koska laajenemisnopeus riippuu inflaation aikaisesta energiatiheydesta, mikä oli hyvin iso.

      Sen jälkeen laajeneminen hidastuu, koska gravitaatio on puoleensavetävä, eli avaruuden eri osat vetävät toisiaan puoleensa.

  9. Jari Toivanen sanoo:

    Pahoittelen, jos tämä kysymys ei ihan täsmälleen asiaan liity. Kerroit tuossa kaksi tarinaa ajasta jutussa, että suhteellisuusteoria on deterministinen, mutta kvanttifysiikka ei ole deterministinen. Voidaanko sanoa jo mitään ennusteita siitä tuleeko kvanttigravitaatio tai muu perustavamman laatuinen teoria olemaan deterministinen vai ei-deterministinen?

    1. Syksy Räsänen sanoo:

      Yleensä ajatellaan, että epädeterministinen, mutta emme tiedä. Nobel-voittaja ’t Hooft on esittänyt malleja siitä, millainen deterministinen fysiikka voisi olla kvanttimekaniikan takana, mutta ne ovat melko lapsenkengissään.

  10. Seppo Lahtinen sanoo:

    Pahoittelen, että vähän raihnaisena vanhuksena kysyn tätä jo vähän myöhässä. Jos valoa nopeamman inflaation jälkeen maailmankaikkeuden (tai havaitsemamme mailmankaikkeuden osan) koko oli yksi sentti, miten se oli 4 minuutin kuluttua 20 valovuotta? Sehän tarkoittaa valoa nopeamman laajenemisen jatkumista.

    1. Syksy Räsänen sanoo:

      Kysymykseen on vastattu aiempien kommenttien yhteydessä.

Vastaa

Sähköpostiosoitettasi ei julkaista. Pakolliset kentät on merkitty *


Konservatiivisuuden nokareet

18.11.2018 klo 22.50, kirjoittaja
Kategoriat: Kosmokseen kirjoitettua , Kosmologia

Olen kirjoittanut useista ehdotuksista pimeäksi aineeksi: nynnyt, aksionit, steriilit neutriinot ja heksakvarkit – vaihtoehtoja on tusinakaupalla. Kommenteissa kollegani Tommi Tenkanen on muistuttanut siitä, että gravitaatioaaltokokeiden LIGO ja Virgo havaitsemat vähän Aurinkoa raskaammat mustat aukot kelpaavat nekin pimeäksi aineeksi. Pimeän aineen ei siis välttämättä tarvitse olla hiukkasia, vaikka se onkin suosituin mahdollisuus.

LIGOn ja Virgon Nobel-palkitut gravitaatioaaltohavainnot ovat antaneet nostetta sille vuosikymmeniä pohditulle mahdollisuudelle, että pimeä aine koostuu mustista aukoista. Kyse ei ole niinkään siitä, että kokeet olisivat paljastaneet jotain uutta, mutta se että mustat aukot ovat huomion keskipisteessä innostaa miettimään mitä kaikkea niillä voikaan tehdä.

Nyt tulin itsekin tutkineeksi sitä, että pimeä aine koostuisi mustista aukoista, yhdessä ohjaamani väitöskirjatutkija Eemeli Tombergin kanssa (joka teki suurimman osan työstä). Päädyin aiheeseen kiertotietä, Higgsin kentän ja kosmisen inflaation kautta.

Usein pimeä aine ja kosminen inflaatio esitetään todisteina siitä, että on olemassa hiukkasfysiikkaa Standardimallin tuolla puolen, jotain kenttiä ja niiden hiukkasia joita emme ole vielä löytäneet. Fedor Bezrukov ja Mikhail Shaposhnikov näyttivät kuitenkin vuonna 2007, miten Higgsin kenttä voi ajaa kosmista inflaatiota. Higgsinflaatiosta on sittemmin tullut yksi tutkituimpia inflaatiomalleja. Olen itse parina viime vuonna tutkinut sitä, mitä Higgs-inflaatio voi kertoa meille siitä, mistä aika-avaruus on tehty. Mutta sen lisäksi, että Higgs antaa hiukkasille massat ja voi olla vastuussa rakenteen alkuperästä, se saattaa myös tuottaa pimeän aineen. Tämä olisi konservatiivinen vaihtoehto, jossa selitetään tunnetuilla ainesosilla mahdollisimman paljon ennen kuin otetaan mukaan tuntemattomia osasia.

Maailmankaikkeudessa on pimeää ainetta noin viisi kertaa niin paljon kuin tavallista, atomiytimistä ja elektroneista koostuvaa ainetta. Jos pimeä aine koostuu mustista aukoista, pitää selittää miksi mustia aukkoja on niin paljon. Mustien aukkojen tuottaminen on periaatteessa helppoa: tarvitaan vain tarpeeksi tiheä kasa ainetta, joka romahtaa. Näin käy tähdille, jotka ovat vähintään noin kymmenen kertaa raskaampia kuin Aurinko. Pimeä aine ei kuitenkaan voi koostua tällä tavalla syntyneistä mustista aukoista. Kosmisesta mikroaaltotaustasta näkee, että pimeää ainetta oli yhtä paljon kuin nykyään jo silloin maailmankaikkeus oli 380 000 vuotta nuori. Tuolloin tähtiä ei vielä ollut.

Pitää siis olla jokin muu keino kehittää massaklimppejä. Tässä kosminen inflaatio tulee mukaan kuvaan. Inflaation aikana maailmankaikkeuden laajeneminen kiihtyy, ja inflaatiota ajavan kentän kvanttivärähtelyt venyvät hiukkasfysiikan mittakaavasta kosmisiin pituuksiin. Inflaation loputtua kenttä hajoaa hiukkaskaasuksi, joka perii kentän epätasaisuudet: sinne missä kentän arvo on isompi syntyy enemmän hiukkasia. Tämä selittää maailmankaikkeuden kaiken rakenteen (galaksien, planeettojen, ihmisten) alkuperän.

Inflaation synnyttämät epätasaisuudet ovat tyypillisesti hyvin pieniä, sadastuhannesosan kokoisia. Se ei riitä mustien aukkojen synnyttämiseen suoraan, näin pienten kuprujen tapauksessa pitää mennä galaksien ja tähtien hitaan kehityksen kautta. Epätasaisuuksien koko määräytyy siitä, miten nopeasti Higgsin kenttä (tai mikä kenttä inflaatiosta onkaan vastuussa) muuttuu inflaation aikana. Mitä nopeammin kenttä muuttuu, sitä tasaisempana se pysyy. Isompien epätasaisuuksien aikaansaamiseksi pitää siis kehittää tapa hidastaa kentän muutosta.

Inflaatiokentän muutos kuin mäkeä alas vierivän pallon liike. Pallon vierimisnopeuden määrää mäen jyrkkyys, inflaatiokentän muutosnopeus riippuu siitä, miten se vuorovaikuttaa itsensä kanssa. Jos vuorovaikutuksen voimakkuus muuttuu nopeasti kentän arvon myötä, liike on nopeaa kuin jyrkässä mäessä. Jos kentän vuorovaikutus on melkein riippumaton kentän arvosta, se liikuu hitaasti – tämä vastaa sitä, että mäessä on laakea osuus, joka viettää vain hyvin loivasti.

Kvanttikenttäteoria sanelee, että muut hiukkaset vaikuttavat siihen, miten Higgs vuorovaikuttaa, ja vuosina 2013-2014 oli pantu merkille, että tämän takia voi syntyä laakea alue, jota ehdotettiin muihin tarkoituksiin. Ajattelin, että sitä sitä voisi käyttää mustien aukkojen tuottamiseen. Osoittautui, että idea ei ollut niin omaperäinen kuin kuvittelin. Viime vuonna oli ilmestynyt jo yksi artikkeli, jossa laskettiin, miten Higgsin vuorovaikutuksen laakean alueen avulla saadaan tuotettua suunnilleen Auringon massaisia mustia aukkoja tarpeeksi selittämään pimeä aine. Aiheen tiimoilta. oli myös jokunen muu artikkeli, joissa käytettiin jotain muuta, spekulatiivista, kenttää, ei Higgsiä.

Tarkastelimme kuitenkin Higgs-tapausta edeltäjiämme perusteellisemmin ja huolellisemmin (ja kun sanon ”tarkastelimme”, tarkoitan lähinnä että jatko-opiskelijani Eemeli tarkasteli). Huomasimme, että laakea kohta ei hidasta kenttää tarpeeksi, että mustia aukkoja syntyisi merkittävästi, toisin kuin kollegamme olivat väittäneet. Mutta Eemeli keksi, että kvanttiefektit voivat saada aikaan myös matalan kuopan. Kenttä hidastuu noustessaan kuopasta ylös, ja kuopan syvyyttä säätämällä nopeutta saa laskettua miten paljon haluaa – eli kenttään saa miten isoja klimppejä tahansa.

Huomasimme kuitenkin (ks. edelliset sulut), että mustien aukkojen massan ja maailmankaikkeuden rakenteen välillä on yhteys, koska ne ovat molemmat peräisin Higgsin kentästä. Mitä raskaampia mustat aukot ovat, sitä enemmän galaksien jakauma ja kosmisen mikroaaltotaustan epätasaisuudet eroavat näkemästämme.

Tämän takia pimeä aine ei voi koostua LIGOn ja Virgon näkemistä mustista aukoista: syntyvien mustien aukkojen massan täytyy olla paljon pienempi, alle tonnin. Pienet mustat aukot höyrystyvät nopeasti, kuten Stephen Hawking osoitti. Ei tiedetä, mitä höyrystymisen loppuvaiheilla tapahtuu. Tämän selvittämiseen tarvittaisiin kunnollinen kvanttigravitaatioteoria, jota meillä ei vielä ole. Vaihtoehtoja lienee kaksi: joko musta aukko haihtuu kokonaan tai jäljelle jää Planckin massan (kymmenen mikrogrammaa) painoinen kvanttinokare. Ensimmäisessä tapauksessa mustat aukot olisivat kadonneet nykypäivään mennessä. Jälkimmäisessä tapauksessa pimeä aine voi koostua tällaisista nokareista.

Jos pimeä aine on mustien aukkojen nokareita, niin sitä ei luultavasti koskaan tulla havaitsemaan muuten kuin gravitaation kautta. Kymmenen mikrogrammaa on tähtien tai ihmisten mittakaavassa vähän, mutta hiukkasfysiikassa valtavan paljon, 10^(16) kertaa enemmän kuin tyypillisten nynnyjen massa. Pimeän aineen massatiheys tiedetään havainnoista, joten mitä isompi on yksittäisen nokareen massa, sitä vähemmän niitä on. Nokareita olisi siis 10^(16) kertaa vähemmän kuin tyypillisiä nynnyjä. Niinpä nokareiden törmäyksiä tavalliseen aineeseen tuskin koskaan nähdään, vaikka niiden vuorovaikutukset eivät olisikaan äärimmäisen heikkoja. Nokareet eivät enää säteile eivätkä hajoa, joten taivaalta ei näy mitään niistä tulevaa säteilyä, eivätkä ne vaikuta tähtien kehitykseen. Voisi siis olla hyvin vaikea osoittaa, että juuri ne ovat pimeää ainetta.

Ajatuksen mustien aukkojen nokareista pimeänä aineena esitti jatko-opiskelija Jane MacGibbon vuonna 1987, ehdotuksemme on ensimmäinen, missä niitä tuotetaan Higgsin kentän avulla. Pitipä idea paikkansa tai ei, se osoittaa, että pimeä aine ei vielä vaadi mitään uutta hiukkasfysiikkaa Standardimallin tuolta puolen.

33 kommenttia “Konservatiivisuuden nokareet”

  1. Cargo sanoo:

    Ovatko nuo kvanttinokareet jotain aitoja pistehiukkasia vai mitä niiden sielunelämästä tiedetään? Vaikuttaa aika ad hokkuspokkus -hiukkaselta. Ja millainen autisti alkaa metsästämään joitain teoreettisen aasinsillan mahdollistaman fluktuaation kautta loihdittuja näkymättömiä hiukkasia?

    Mutta eiköhän pari uutta kvanttitermiä Lagrangen tiheyteen tuo valoa tähän pimeyteen 🙂

    1. Syksy Räsänen sanoo:

      Niiden halkaisija on Planckin mittakaavan luokkaa, eli noin 10^(-34) m.

      Toivon, että tällä palstalla esitetyt kommentit ovat jatkossa asiallisia. Muuten niitä ei julkaista. Toistuvasti asiattomia kommentteja lähettäville tulee pysyvä kielto.

  2. Juha sanoo:

    Hei. En ole varma olinko kärryillä aiempien tekstien kanssa, mutta eikö pimeän aineen pitäisi kasautua ”huonommin” kuin tavallinen aineen? Miten tämä sopii yhteen tuon musta-aukko selityksen kanssa?
    Aimepi lainaus:
    ”Ajatus siitä, että pimeä aine olisi osa hiukkasfysiikan sektoria, joka on samanlainen kuin näkemämme, mutta vain meiltä piilossa, ei ole uusi. Se ei voi pitää paikkaansa, koska havaittu pimeän aineen klimppiytyminen on erilaista kuin näkyvän aineen. Näkyvä aine menee pimeää ainetta enemmän kasaan, koska se voi jäähtyä sähkömagneettisten vuorovaikutusten takia.”

    1. Syksy Räsänen sanoo:

      Pienistä mustista aukoista koostuva kaasu ei voi jäähtyä sähkömagneettisten vuorovaikutusten kautta, sen enempää kuin heikosti vuorovaikuttavista hiukkasista koostuva kaasu.

      Tavallinen aine jäähtyy muodostamalla molekyylejä, jotka säteilevät energiaa pois.

  3. Jernau Gurgeh sanoo:

    Kuinka hyvin pienet mustat aukot sopivat Luotijoukosta (Bullet Cluster) saatuihin havaintoihin?

    1. Syksy Räsänen sanoo:

      Kirjoitin luotiryppäästä täällä:

      https://www.ursa.fi/blogi/kosmokseen-kirjoitettua/luodin-jaljet/

      Havainnot luotiryppäästä rajoittavat sitä, miten vahvoja vuorovaikutuksia pimeän aineen osasilla voi olla toistensa kanssa gravitaation lisäksi.

      Pienillä mustilla aukoilla vuorovaikuttavat toistensa kanssa käytännössä vain gravitaation kautta, eli ne sopivat havaintoihin erinomaisesti.

  4. Erkki Tietäväinen sanoo:

    Tämä saattaa olla tyhmä kysymys, mutta kysyn kuitenkin: Eikö mustan aukon ominaisuuksiin kuulu aina riittävän suuruinen massa ja painovoima, jolla se estäisi myös valon ”karkaamisen”? Miten kymmenen mikrogramman painoiseksi kutistunut mustan aukon nokare (kvanttinokare) voi olla painovoimaltaan niin suuri, että se pysyy mustana aukkona ja toimii selittäjänä pimeälle aineelle ?

    1. Syksy Räsänen sanoo:

      Mustien aukkojen painovoima on sama kuin minkä tahansa muunkin samanmassaisen kappaleen. Jos Aurinko puristuisi mustaksi aukoksi, se ei vaikuttaisi planeettojen ratoihin.

      Mustien aukkojen massa voi olla miten pieni tahansa (oletettavasti vain tuohon Planckin massaan asti), oleellista on se paljonko massaa on niiden säteen sisällä.

      1. Erkki Tietäväinen sanoo:

        Vastauksen alkuosan yhteyttä tekemääni kysymykseen en ymmärrä.
        Itse kysymykseeni palatakseni on pakko tehdä jatkokysymys, johon toivon vastauksen, jonka maallikkokin tajuaa: Kymmenessä mikrogrammassa on massaa kymmenen mikrogrammaa riippumata siitä, miten suuri tai pieni on mustan aukon nokareen säde. Miten siis niin kevyt kappale voi toimia mustan aukon tavoin?

        1. Syksy Räsänen sanoo:

          Kuten yllä kirjoitin, oleellista ei ole massa, vaan se, miten pienen säteen sisällä se on.

          Miten pienestä tahansa määrästä ainetta tulee musta aukko, kunhan sen puristaa tarpeeksi pieneksi.

          1. Eusa sanoo:

            Suomennetaanko lisää?

            Musta aukko nousee matematiikasta, kun gravitoiva energiatiheys nousee paikallisesti riittävän suureksi.

            Musta aukko on matemaattinen olio, kunnes siitä saadaan fysikaalinen havainto. Luontohan voisikin lainalaisuuksissaan estää tapahtumahorisontin muodostumisen, vaikka harventamalla tilaa sisäisesti dr. Whon Tardis-aluksen tapaan, vrt. Shapiro…

            Siksi on tärkeää, että ponnistellaan havaintojen saamiseksi, mm. EHT-projektissa, jossa pyritään ottamaan valokuva tapahtumahorisontista.

  5. pekkap0 sanoo:

    Voiko aika-avaruudella olla hitaus/inertia, jolloin Luotiryppään kaasujen aiheuttamat gravitaatiokuopat jatkaisivat törmäystä edeltäneisiin liikesuuntiin? Hitauden voisi ehkä myös kuvailla aika-avaruuden vuorovaikutuksena itsensä kanssa.

    1. Syksy Räsänen sanoo:

      Kysymys menee sen verta ohi merkinnän aiheesta, että ei siitä tässä sen enempää.

  6. Planckin massaisen mustan aukon maksimaalinen impulssimomentti (mPlanck*c*lPlanck) on luokkaa Planckin vakio. Herää kysymys mitä tapahtuu jos aukon impulssimomentti on puolilukuinen, eli jos meillä on fermioninen miniaukko. Bosoninen (pyörimätön, siis spin nolla) Planckin massainen aukko voi käsittääkseni hajota esimerkiksi kahdeksi fotoniksi, mutta ei liene yhtä selvää miten fermioninen Planckin massainen aukko voisi hajota. Vai onko? Kysymys taitaa palautua siihen että säilyttääkö aukko leptoniluvun kaltaisen ominaisuuden vai pitääkö no hair -teoreema ottaa kirjaimellisesti myös mikroskooppisille aukoille.

    Ajatus olisi siis että jossain vaiheessa alkuräjähdystä sekä bosonisia että fermionisia pieniä aukkoja olisi muodostunut, vieläpä eri kokoisia, mutta bosoniset aukot olisivat sittemmin haihtuneet säteilyksi ja fermionisetkin melkein, paitsi että niistä olisi jäänyt Planckin massainen hajoamaton jäännös.

    1. Syksy Räsänen sanoo:

      Planckin skaalan tapahtumista ei tiedetä mitään varmaa.

      1. Nii-in. Entä jos tehdään hypoteesi että Planckin epookissa syntyi stabiileja Planckin massaisia aukkoja (ottamatta kantaa mekanismiin), voisiko kyseisiä aukkoja olla inflaation jälkeen vielä riittävän tiheässä selittämään pimeän aineen?

        1. Syksy Räsänen sanoo:

          Ei, koska inflaatio pyyhkisi ne pois.

          Yksi inflaation alkuperäinen motivaatio 80-luvulla oli juuri se, että se siivoaa kaiken aiemman pois. Yksi esimerkki olin silloin suosiossa olleiden yhtenäisteorioiden monopolit, jotka käyttäytyvät oleellisesti samalla tavalla kuin pienet mustat aukot mitä pimeään aineeseen tulee. Ongelmana oli se, että niitä laskettiin syntyvän liikaa yhtenäisteorioiden olomuodon muutoksissa.

          1. Onhan se vuorovaikutus tosiaan varsin heikko, esimerkiksi jos on neutroni yhden femtometrin päässä Planckin massaisesta aukosta, niiden välisen gravitaatioattraktion energia on vasta 1.5e-11 eV, eli vastaa alle mikrokelvinin lämpötilaa.

            Asiaa voi kuitenkin katsoa vähän toisestakin näkökulmasta. Jos massallinen hiukkanen on aukosta vaikka kahden ångströmin päässä (tyypillinen atomien välimatka kiinteässä aineessa), sen kokema gravitaatiokiihtyvyys on ihan mukavalta kuulostava eli rakettimainen 3.7g. Viidentoista nanometrin päässä kiihtyvyys on sama kuin Auringon gravitaatiokenttä Maan etäisyydellä eli 0.6 milligeetä. Ongelma on vielä se että aukot yleensä viilettävät nopeasti ohi eivätkä liiku samaa nopeutta kuin laboratorio, paitsi jotkut harvat. Kuitenkin, jäähdyttämällä aineet ja menemällä avaruuteen, en sulkisi pois mahdollisuutta että tuosta saattaisi jonain päivänä jotain mittausmahdollisuutta irrota.

  7. Mustan aukon haihtumisnopeus voittaa kasvamisen kun sen Hawkingin säteily on ympäristöä kuumempaa. Kun primordiaaliset aukot pienentyvät haihtumalla, ne siis kontribuoivat ympäristöään kuumempaa, ei-termistä säteilyä. Eli aukot loistavat kuin mikrotähdet ja kuumentavat ainetta uudelleen, vähän samoin kuin myöhemmin ensimmäiset tähdet ionisoivat kaasua uudelleen. Meneekö tämä aukkojen haihdunta ajallisesti päällekkäin Higgsiin liittyvän reheating-vaiheen kanssa, ja pystyykö aukkojen ja Higgsin suhteellisia kontribuutioita lämmitykseen vertaamaan kvantitatiivisesti?

    1. Syksy Räsänen sanoo:

      Mustien aukkojen säteily on termistä. Niiden ja ympäristön lämpötilan suhde riippuu aukon massasta ja ympäristön lämpötilasta: mustat aukot voivat olla kuumempia tai kylmempiä kuin ympäristö. (Esimerkiksi nykyiset Auringon massaiset mustat aukot ovat paljon ympäristöä kylmempiä.)

      Jos kyse on tapauksesta, missä pimeä aine on mustia aukkoja, niin ne muodostuvat reheatingissa syntyneestä aineesta ja tyypillisesti haihtuvat kauan reheatingin jälkeen.

  8. Lentotaidoton sanoo:

    Räsänen: Jos pimeä aine on mustien aukkojen nokareita, niin sitä ei luultavasti koskaan tulla havaitsemaan muuten kuin gravitaation kautta. Kymmenen mikrogrammaa on tähtien tai ihmisten mittakaavassa vähän, mutta hiukkasfysiikassa valtavan paljon, 10^(16) kertaa enemmän kuin tyypillisten nynnyjen massa. Pimeän aineen massatiheys tiedetään havainnoista, joten mitä isompi on yksittäisen nokareen massa, sitä vähemmän niitä on. Nokareita olisi siis 10^(16) kertaa vähemmän kuin tyypillisiä nynnyjä. Niinpä nokareiden törmäyksiä tavalliseen aineeseen tuskin koskaan nähdään, vaikka niiden vuorovaikutukset eivät olisikaan äärimmäisen heikkoja. Nokareet eivät enää säteile eivätkä hajoa, joten taivaalta ei näy mitään niistä tulevaa säteilyä, eivätkä ne vaikuta tähtien kehitykseen. Voisi siis olla hyvin vaikea osoittaa, että juuri ne ovat pimeää ainetta.

    10 mikrogrammaa on hiukkasfysiikassa valtavan paljon. Kun kuitenkin WIMPejäkin (ja julmasti kevyempiä axionejakin) tietysti metsästetään, niin eikö periaatteessa myös älytömän julmasti raskaampia ”nokareitakin” voisi, vaikka niitä vastaavasti olisi häviävästi harvemmassa.

    Osaatko kuvitella mitään metodia (joko standarditeorian puitteissa tai ulkopuolisten mallien puitteissa) joissa voisi edes teoreettisesti osoittaa niiden olevan pimeää ainetta (tai että niitä ylipäätään on olemassa)? Potku on potku vaikka niitä olisi harvassakin.Ymmärrän kuitenkin että nykykeinoin asia on (ehkä) mahdoton. Vai onko peli kokeellisesti menetetty?

    1. Syksy Räsänen sanoo:

      Nokareiden vuorovaikutuksia muiden hiukkasten kanssa ei tunneta, mutta oletettavasti ne ovat erittäin heikkoja.

      Mieleeni ei tule mitään tapaa käytännössä havaita niitä nähtävissä olevan teknologian avulla muuten kuin gravitaation kautta.

      1. Voisiko ohikulkeva musta aukko aiheuttaa jotain harvinaisia leptoni- tai baryonilukua rikkovia prosesseja, jos pätee että aukon ainoat parametrit ovat massa, pyörimismäärä ja sähkövaraus? Esim. että yksi protonin kvarkeista putoaa ohikulkevaan aukkoon ja tulee hetken päästä ulos sieltä Hawkingin säteilyn fotonina.

        1. Syksy Räsänen sanoo:

          Nokareiden fysiikkaa ei juuri tunneta, mutta niiden lukumäärätiheys on niin pieni (Maapallon kohdalla noin yksi per kymmenentuhatta kuutiokilometriä), että niiden ei-gravitaatio-vuorovaikutuksen havaitseminen vaikuttaa toivottomalta.

  9. Eusa sanoo:

    https://arxiv.org/pdf/1810.12218

    Primordiaalisten kuu-kokoluokan kokkareiden observointi-ideointia.

  10. Lentotaidoton sanoo:

    Räsänen: ”tai jäljelle jää Planckin massan (kymmenen mikrogrammaa) painoinen kvanttinokare”.

    ”Kvanttinokare” on vähän epämääräisesti sanottu (ei ainakaan kuullosta minun korvaan kovin tieteelliseltä). Voitko selventää? Todennäköisesti on kuitenkin kysymys baryonisesta (?) aineesta, vai mistä? (eli minkä kentän ”nokare?). Kun kerta toinen vaihtoehto on ”haihtuu kokonaan”.

    1. Syksy Räsänen sanoo:

      Kyseessä ei ole baryoninen aine, vaan pienet mustat aukot.

  11. Lentotaidoton sanoo:

    Andrei Linden kaoottisessa inflaatiomallissa havaittava maailmankaikkeutemme olisi tullut halkaisijaltaan 10^-35 metrin eli Planckin pituuden suuruisesta hipusta, johon yhtyi erittäin suuri energiatiheys noin 10^94 grammaa kuutiometrissä tuottaen kokonaismassan 10^-5 grammaa eli Planckin massan, eli saman ”pölyhiukkasen” massan. Ovatko nämä luvut vain yhteensattuma? Vai onko ero kuumuuden valtavassa energiassa?

    1. Syksy Räsänen sanoo:

      Molemmissa on kyseessä Planckin mittakaava (oli se sitten tiheys, pituus tai massa, ks. https://www.ursa.fi/blogi/kosmokseen-kirjoitettua/kaymattomista-korpimaista-vihoviimeinen/), jossa kvanttigravitaatiota ei enää voi jättää huomiota ja tullaan ymmärryksemme rajalle.

Vastaa

Sähköpostiosoitettasi ei julkaista. Pakolliset kentät on merkitty *


Miksi on turha etsiä seuraavaa Einsteinia

13.11.2018 klo 15.31, kirjoittaja
Kategoriat: Kosmokseen kirjoitettua , Kosmologia

Puhun Tieteen päivillä lauantaina 12. tammikuuta 2019 kello 14.30 Porthanian salissa PI luentosarjassa Rohkeaa tutkimusta luonnontieteissä otsikolla Rohkeus ja rakenteet: miksi on turha etsiä seuraavaa Einsteinia. Puheen tiivistelmä on seuraava (Tieteen päivien kirjassa on myös artikkelini aiheesta.)

Tänä vuonna tulee kuluneeksi sata vuotta siitä, kun kokeellisesti osoitettiin, että Auringon gravitaatio taivuttaa valonsäteitä. Havainto vahvisti yhden yleisen suhteellisuusteorian keskeisistä ennustuksista ja teki Albert Einsteinista maailman ensimmäisen tiedejulkkiksen.

Käsitys Einsteinista yksinäisenä nerona ja tieteilijän perikuvana on sittemmin riivannut tiedettä. Sikäli kun kuva pitää paikkansa, se on poikkeus. Siltä osin kun se ei pidä paikkaansa, se on huomattavasti opettavaisempi.

Yleisen suhteellisuusteorian, maailmankaikkeuden laajenemisen ja pimeän aineen ja maailmankaikkeuden laajenemisen tutkiminen halki vuosisadan osoittaa, millaista tieteen tekeminen on. Parhaatkin tutkijat ovat toisinaan täysin väärässä, edistys syntyy tutkijoiden tiiviistä vuorovaikutuksesta, ja havainnot ovat korvaamattomia sen selvittämisessä, mikä on totta ja mikä vain toivetta.

Ajatus tieteestä yksinäisten nerojen pelikenttänä vääristää yhä käsitystä tieteestä, niin tutkijoiden kuin rahoittajien parissa. Tutkimuksen vaaliminen on pitkäjänteinen prosessi, jossa pitää nostaa koko tutkijayhteisön valmiuksia ja luoda ympäristö, joka helpottaa myös vaikeiden, aikaa vievien ja kiistanalaisten ideoiden tutkimista. Sen sijaan yhä suurempi osa rahoituksesta on kilpailtua, ja tutkimusrahoitusta haettaessa tutkijoiden odotetaan lupaavan mahdollisia läpimurtoja ja liioittelevan tutkimuksen merkitystä. Mikä vaikutus tieteeseen on sillä, että tutkijoita ohjataan harkittuun ja järjestelmälliseen epärehellisyyteen asiassa, joka on keskeinen uralla etenemiselle?

Päivitys (19/01/19): Puhun samasta aiheesta myös Oulun ja Mikkelin Tieteen päivillä, ajat ja paikat täällä.

6 kommenttia “Miksi on turha etsiä seuraavaa Einsteinia”

  1. Tässä saattaa olla merkittäviä alakohtaisia eroja. Oma kokemukseni on nimittäin suunnilleen päinvastainen, sikäli kun näissä ”pehmeissä” asioista vastakohdista on mielekästä puhua. Koen että maailma on siinä mielessä pieni että ”einsteineja” jotka vievät sitä eteenpäin on vähän, ehkä sama määrä kuin Einsteininkin aikana. Koen myös että rahoitus ohjautuu hyvin vahvasti vanhan viilaamiseen uuden keksimisen sijasta. Esimerkiksi paljon tutkimusrahaa käytetään vetytalouden moninaisten teknisten haasteiden voittamiseen, mutta juuri yhtään ei siihen että ammoniakkia käyttämällä niitä ongelmia ei tarvitsisi edes ratkaista. Vain yksi esimerkki.

    Mutta kuten sanottu, alakohtaiset erot saattavat olla suuria. Jos ajattelen asiaa teoreettisen fysiikan ja kokeellisen hiukkasfysiikan näkökulmasta, luullakseni pystyn kuvittelemaan mitä kautta olet johtopäätökseen tullut. Molemmat taitavat olla aloja joissa työ lienee usein systemaattisenoloista puurtamista ja läpimurrot usein syntyvät sen puurtamisen pohjalta.

  2. Cargo sanoo:

    Kai nyt edes Newtonia saa kutsua ”yksinäiseksi neroksi”, kun kehittää toimivan painovoimateorian sellaisen pähkähullun ajatuksen pohjalta, että voima voi välittyä ilman suoraa kosketusta?

    Mitä tähän päivään tulee, niin esim. Brian Greene tekee tieteelle karhunpalveluksen, kun maalaa täysin polarisoituneen kuvan tieteen eturintamasta. Sellainen Hollywood-meininki keskittää suuren yleisön huomion vain säikeisiin ja sankaritarinoihin, vaikka teknologisia ongelmia ratkovat tieteilijät olisivat juuri niitä, joita ihmisten tulisi lumoutuneina kuunnella. Asian toki ymmärtää, sillä nykymaailmassa tiede on ottanut uskonnon roolin.

    Feynman sanoi, että Einstein ei saanu uransa toisella puoliskolla mitään aikaiseksi, koska lakkasi pohtimasta fysiikan perusilmiöitä. Ja kun ottaa huomioon tieteen nykytilan, niin itse suuntaisin rahoitusta matemaattisesta fantasiakirjallisuudesta kohti uusien teknologioiden kehittämistä.

    1. Syksy Räsänen sanoo:

      Juurikin Newton on tullut tunnetuksi siitä, että hän korosti muiden tutkijoiden merkitystä: ”Jos olen nähnyt muita kauemmas, se on siksi, että olen seissyt jättiläisen olkapäillä.”

      Mutta tutkimuksen tekeminen Newtonin aikaan oli niin erilaista kuin nykyään, että hänestä keskusteleminen suuntaan tai toiseen ei juuri valaise nykytilannetta.

Vastaa

Sähköpostiosoitettasi ei julkaista. Pakolliset kentät on merkitty *


Fantasiakirjallisuuden arvioimista

30.10.2018 klo 13.21, kirjoittaja
Kategoriat: Kosmokseen kirjoitettua

Syyskuun lopussa jätin apurahahakemuksen Suomen Akatemialle, kuten noin 3 000 muutakin tutkijaa. Kaikista tieteilijän toimenkuvaan kuuluvista asioista –tutkimus, opetus, popularisointi, vertaisarviointi, hallinto, konsultointi– apurahojen hakeminen on turhauttavinta. Tutkijat usein valittavat siihen menevästä ajasta ja vaivasta. Yksi syy on se, että hakemusten laatimiseen käytetyt tunnit menevät luultavasti hukkaan, koska vain pienelle hakemuksista myönnetään rahaa. Mutta oman kokemukseni mukaan hakemusten tekeminen on raskasta myös siksi, että niiden kirjoittamisen ja muun tieteellisen työn välillä on sisällöllinen, ei vain ajallinen, jännite.

Apurahoja on erilaisia: yksittäisiä konferenssimatkoja tukevia, jatko-opiskelua vuoden kerrallaan tukevia, ja vuodesta kolmeen vuoteen väitelleen tutkijan työskentelyä tukevia. Keskityn tässä monivuotisiin hankeapurahoihin, joista Akatemian hankeapurahat ovat tyypillinen esimerkki. Niitä haetaan vuotta etukäteen neljäksi vuodeksi, ja apuraha kattaa suunnilleen hakijan jatko-opiskelijan palkan sekä matka- ja muita sekalaisia kuluja väitöskirjatyön ajaksi (tai väitelleen tutkijan palkan lyhyemmälle ajalle). Hiukkaskosmologian alalla tämä tarkoittaa sitä, että hanke rahoittaa tutkimusta kolmen artikkelin verran – oikeastaan rahoitus on vain osittaista, koska jatko-opiskelija ei tee tutkimusta yksin vaan yhteistyössä muiden kanssa.

Tästä huolimatta hakemukset pitää laatia kuin hankkeessa olisi kyse merkittävästä harppauksesta, ei vain tavallisesta tutkimuksesta, joka etenee pienin askelin ja odottamattomiin suuntiin. Ristiriitaisesti samalla hakemuksiin kuitenkin pitää laatia tarkat askelmerkit etenemisestä vuosi vuodelta ja aikataulu tulevista tuloksista. Ei siis riitä, että pitää suurennella tutkimuksen merkitystä, vaan pitää myös sepittää sen yksityiskohtainen kulku. Todellisuudessa ei voi tietää viittä vuotta etukäteen miten kaikki tulee sujumaan, ellei mistään muusta syystä, niin siksi, että maailmassa on tuhansia muitakin tutkijoita, joiden löytöjä ja oivalluksia ei kukaan pysty etukäteen arvaamaan.

Kyse on eräänlaisesta fantasiakirjallisuudesta, jossa sekä kirjoittaja että lukija tietävät, että teksti ei ole totta, mutta ihanteena on saavuttaa suspension of disbelief, tila, jossa pystyy laittamaan syrjään sen, että kyseessä on kuvitelma.

Kun tehdystä tutkimuksesta raportoidaan tieteellisissä artikkeleissa, ihanteena on asioiden esittäminen mahdollisimman totuudenmukaisesti, mihin kuuluu työn laittaminen oikeisiin raameihin ja sen puutteiden ja rajoitusten läpikäyminen. Apurahahakemukset ovat tyystin toisenlainen ilmaisun muoto, joka muistuttaa sijoittajien houkuttelemista: niissä myydään tavallista tutkimusta mahdollisena läpimurtona jo ennen kuin sitä on tehty. Luultavasti usein hakemuksissa markkinoitua tutkimusta ei lopulta edes tehdä, koska vain tutkimus voi osoittaa, mitkä tutkimuskysymykset ovat kiinnostavia ja mihin suuntaan kannattaa edetä. (En tiedä millaista riippumatonta seurantaa tästä on tehty, olisi mielenkiintoista lukea tutkimus siitä, miten hakemukset ja toteutunut tutkimus suhtautuvat toisiinsa.)

Apurahat eivät ole erityistä panostusta yliopiston perustehtävien päälle, vaan niillä rahoitetaan myös yliopiston perustoimintaa. Esimerkiksi yliopiston tutkijoiden velvollisuuksiin kuuluu jatko-opiskelijoiden ohjaaminen, ja heitä rahoitetaan usein ohjaajien apurahoilla.

Tilanteen tekee entistä ongelmallisemmaksi se, että hakemusten arvioimisessa on paljon sattumanvaraisuutta. Tämän näkee siitä, miten peräkkäisinä vuosina sama hakemus voi saada aivan erilaisia arvioita: yhtenä vuotena heikkoutena pidetyt piirteet voidaan seuraavana vuonna lukea vahvuudeksi (ja hakemus saada rahoituksen), tai päin vastoin.

Eräs kollegani kertoi, että kun hän haki arvostettua European Research Councilin (ERC) apurahaa, kielteisen arvion perusteluna käytettiin mm. sitä, että suuri osa hänen tutkimuksestaan on julkaistu osana kansainvälistä IceCube-tutkimusryhmää, eikä siis kerro hänen omasta osaamisestaan. Mielenkiintoinen peruste ottaen huomioon, että hakijalla ei itse asiassa ollut ainuttakaan paperia IceCuben kanssa. (Henkilö lähti sittemmin Euroopasta australialaisen yliopiston fysiikan laitoksen johtajaksi.) Mainittakoon, että ERC-hakemusten tekemiseen suositellaan käyttämään vähintään kaksi viikkoa täysipäiväistä työaikaa. Myös akatemiahakemuksen tekemiseen kuluu helposti työviikko, ja hakemuksista on tullut entistä raskaampia vuosien varrella. Tuo ERC-tapaus oli poikkeuksellisen räikeä, mutta selvät faktavirheet arvioissa eivät ole harvinaisia.

Arvioijat ovat tiedeyhteisön jäseniä, joilla ei yleensä ole mitään koulutusta hakemusten arviointiin. Helsingin yliopiston kolmevuotisten apurahojen arviointilautakunnassa istuneena voin todeta, että siinä ei myöskään tarjottu mitään opastusta esimerkiksi omien ennakkoluulojen huomioimiseen arvioinnissa, eikä Akatemiankaan ohjeissa näytä sellaista olevan. Tämä on tavallista: ainoa omalle kohdalleni sattunut poikkeus on Iso-Britannian Royal Society, jonka arvioijille lähetettävään materiaaliin kuuluu teksti- ja videoperehdytys tiedostamattomien ennakkoluulojen huomioimiseen. Tällaisen puute haittaa erityisesti naisia, koska heihin kohdistuu haitallisia ennakkoluuloja.

Apurahaprosessin satunnaisuus ja ongelmat kasautuvat, koska myönnettyjä apurahoja voidaan käyttää kriteerinä seuraavien apurahojen myöntämisessä. Lisäksi ongelmat leviävät muualle tieteelliseen toimintaan, koska apurahojen saamista käytetään tutkijoiden laadun mittarina. Helsingin yliopiston matemaattis-luonnontieteellisen tiedekunnan dekaani Kai Nordlund onkin todennut, että rahoituksen saaminen on eräs tärkeimpiä meriittejä tieteellisellä uralla etenemiseen, ja se on usein ratkaiseva tekijä professuureja ja muita pysyviä työpaikkoja täytettäessä silloin kun hakijat ovat muuten saman tasoisia. Erityisesti isojen ulkomaisten apurahojen, kuten ERC:n apurahojen, saaminen on noussut yhä tärkeämmäksi. (Sanottakoon, että Helsingin yliopiston fysiikan osasto on menestynyt hyvin apurahojen hakemisessa niin Suomen Akatemialta, ERC:ltä kuin muualtakin.)

Apurahahakemusten laatimisessa on hyvätkin puolensa: siinä pysähtyy miettimään tutkimustaan laajemmasta näkökulmasta ja miettineeksi suuntaa tavallista pidemmälle. Tämä ei kuitenkaan kumoa niitä ongelmia, mitä aiheutuu siitä, että yliopistojen toimintaa, mukaan lukien perustoimintaa, rahoitetaan yhä enemmän kilpaillulla rahoituksella.

Sen lisäksi, että hakemusten tekemiseen haaskaantuu vuosittain kymmeniä tuhansia työpäiviä, sopii kysyä, mikä vaikutus tutkimuksen kenttään on sillä, että tutkijoita koulitaan ajattelemaan tiedettä tavalla, joka näyttää hyvältä hakemuksissa, joiden kriteerit eivät vastaa tutkimuksen todellisuutta ja lupaamaan läpimurtoja hitaamman, perusteellisemman ja arvaamattomamman tutkailun sijaan.

On myös kyseenalaista, onko empiiristä näyttöä siitä, että kilpailutus parantaa tutkimuksen tasoa. Suomi muuten sattuu erään tutkimuksen olemaan malliesimerkki kilpaillun rahoituksen osuuden ja tutkimuksen tehokkuuden välisestä negatiivisesta korrelaatiosta. Tosin, kuten kirjoittajat itsekin toteavat, tarvittaisiin enemmän dataa luotettavien johtopäätösten vetämiseen, mutta heidän arvioonsa ”most of science policy is hardly evidence based” (suurin osa tiedepolitiikasta tuskin perustuu todistusaineistoon) on helppo yhtyä.

Kasvavan kilpailun vaihtoehto on yksinkertainen: kilpaillun rahoituksen osuuden pienentäminen. On erilaisia ideoita siitä, miltä täysin erilainen rahoitustapa näyttäisi, ja niiden hyötyjä ja haittoja olisi syytä pohtia vakavasti. Vaatimaton askel oikeaan suuntaan olisi jatko-opiskelijoiden rahoituksen siirtäminen enimmäkseen kilpaillusta rahoituksesta enimmäkseen yliopistojen tohtoriohjelmien rahoitukseksi ja muun perustoiminnan rahoittaminen vakaalla rahoituksella.

Kilpaillun osan rahoitusta arviointia tulisi lisäksi parantaa: jos arvioita on tarkoitus käyttää tieteellisen laadun arvioimiseen, niiden pitäisi olla toistettavia, arvioijat pitäisi perehdyttää työhönsä asianmukaisesti ja kannustimia pitäisi muuttaa siten, että hakemusten sisältö vastaa tutkimuksen todellisuutta, sen sijaan että olisi irrallinen fantasiamaailma.

Päivitys (01/11/18): Kirjoitus on julkaistu uudelleen Yliopistokäänteen sivuilla, ja Suomen Akatemian tiedeasiantuntija Vera Mikkilä on kommentoinut sitä Twitterissä.

16 kommenttia “Fantasiakirjallisuuden arvioimista”

  1. Hyvin sama kokemus minulla. Vaikeampi sen sijaan sanoa mitä asialle voisi tehdä.

    Tiede ylipäätään näyttää olevan ihmiskunnalle vaikeaa. Joskus 1800-luvulla tiede oli vaikeaa, koska sitä pystyivät tekemään vain lordit, ja heitä oli vähän. Nykyään kun tutkijoita on paljon, ongelmat ovat muunlaisia. Kuljetaan kapeaa polkua. Sen soveltavalla puolella tiedettä uhkaavat yritysten ja poliitikkojen lyhyen tähtäimen intressit, ja teoreettisella laidalla vaanii dogmatismi. Joskus ne jopa liittoutuvat ja polku on siitä kohdasta poikki.

  2. Kari O. sanoo:

    ”Ristiriitaisesti samalla hakemuksiin kuitenkin pitää laatia tarkat askelmerkit etenemisestä vuosi vuodelta ja aikataulu tulevista tuloksista. ”

    Tuo kuulostaa aika pahalta.. ikään kuin halutaan varmistaa se, ettei mitään merkittäviä uusia tieteellisiä löytöjä tehdä. Jos siis tutkimuksen tulokset pitää noin tietää jo etukäteen, niin mitä uutta tai arvokasta tieteellistä tietoa siinä enää voi syntyä?

    Toki joskus on niin, että tutkijalla voi olla jokin idea tai ”hunch” mielessään jostakin joka sitten pitää vain työstämällä työstää näkyväksi julkaisun tms. muodossa.

    Toisaalta, historiassa on lukuisia esimerkkejä siitä, miten merkittävät ja käänteentekevät tieteelliset löydöt jotenkin karttavat niitä tahoja, joilla on suurin rahoitus.

  3. AK sanoo:

    Tutkimussuunnitelman laatiminen voi tosiaankin olla hyödyllistä. Suunnitelmasta poikkeaminen on enemmän sääntö kuin poikkeus – riippuu toki aiheesta, joskus samaa alunperin määriteltyä tietä voi kulkea vuosikymmeniä. Asiantunteva arvioija määrittelee suunnitelman uskottavuutta, erinomaisuutta ja vaikuttavuutta (mikä perustutkimuksen kohdalla ei ole kovin helppoa).

    Kilpaillun rahoituksen osuutta voisi vähentää perustutkimuksessa ja erityisesti aloilla, joilta arvioitavaksi tulee jatkuvasti runsaasti erinomaisia hakemuksia.

    Mikä on paras tapa valita rahoituksen saaja?

    1. Syksy Räsänen sanoo:

      On tuskin olemassa yhtä parasta tapaa, ja erilaisiin rahoituksiin sopivat erilaiset käytännöt.

      Esimerkiksi jatko-opiskelijoiden rahoittaminen olisi käytännöllisintä järjestää etupäässä yliopiston tohtoriohjelmien kautta. Jatko-opiskelu-oikeutta pitää hakea jo nyt (ja senkin hakemuksesta tulee koko ajan raskaampi), asia hoituisi sen arvioimisen yhteydessä. Ja sen arvioimisen olisi parasta tehdä nykyistä lähempänä väitöskirjatyön tekijöitä, ei korkeammalla hallinnon tasolla.

      Kysymys jatko-opiskelupaikkojen määrästä eri aloilla on sitten oma kysymyksensä.

      En ole tutustunut rahoitusmallien kirjoon, merkinnän loppupuolella mainitsemassani esimerkissä https://www.sciencemag.org/careers/2014/01/new-funding-model-scientists on omat hyvät puolensa ja omat ongelmansa, jälkimmäiset etenkin silloin kun rahaa jaetaan pienessä piirissä.
      Merkinnän lopussa mainitsen

  4. Olen hakenut apurahaa oppimateriaalin tekemiseen animaatioista. Sellaista ei suomenkielisenä ole. Kun aloitin opettamisen 2011, jouduin tekemään kaikki materiaalit alusta saakka itse ja tilanne on pysynyt samanlaisena nyt seitsemän vuotta. Kun hain apurahaa, sain asiasta päättävän tahon langanpäähän. Hän oli kyllä kiinnostunut asiasta ja tieto siitä, että opetusmateriaaleja ei ole, tuli hänelle yllätyksenä. Keskustelun lopuksi hän muotoili asian minulle näin:”Hyvä on, oppimateriaaleja ei ole. Mutta kuka sanoo, että JUURI SINÄ olisit oikea henkilö niitä tekemään?”. Mielestäni tämä kommentti kertoo paljon siitä, kuinka peli pyörii. Olennaista ei ole se, että on kulttuurissa/tieteessä on ilmiselvä tutkimuskysymys, jota kuuluisi tutkia, tai kulttuurissa palvelutarve, joka kuuluisi täyttää. Vaan KUKA sille alueelle saa mandaatin mennä. Ja kenellä on valta päättää asiasta. Laatukriteeriksi ei riitä alan koulutus, työkokemus eivätkä näytöt. Vaan joku muu määrittelemätön tekijä, joka on yksin päättävän tahon tiedossa. – Nyt asiasta on kulunut tasan vuosi, enkä ole jaksanut hakea apurahaa uudelleen. Sama henkilöhän siellä olisi taas jakelemassa viisauksiaan. Oppimateriaalia aiheesta ei edelleenkään ole, sitä ei ole kukaan tuottanut eikä rahoittanut. Omat oppimateriaalini olen antanut jakoon oppilailleni ja osin myös verkkoon. Eikä tämä alue ole suinkaan ainoa, josta olen kerryttänyt kokemusta Absurdaniasta nimeltä Apurahamaa.

  5. Erkki Kolehmainen sanoo:

    Nyt Syksy Räsänen kirjoitti asiaa, mutta ei mielestäni tuonut riittävästi esiin sitä, mikä vaikutus kilpaillulla rahoituksella on tieteen itsensä kehitykseen. Otanpa asiasta esimerkin Syksyn omalta alalta. Hakija 1 esittää hakemuksessaan kehittävänsä menetelmän, jolla pimeän aineen olemassaolo todistetaan. Hakija 2 puolestaan esittää suunnitelmassaan osoittavansa, ettei pimeää ainetta ole. Ottaen huomioon sen tosiasian, että ko. alan asiantuntijoista valtaosa uskoo pimeän aineen olemassaoloon, niin on selvää, että hakijan 1 hakemus saa myönteisen palautteen, mutta hakijan 2 suunnitelma torpataan epärealistisena. Näin siis kilpailtu rahoitus varmistaa sen, ettei paradigman muutosta tieteessä voi tapahtua. Jos tutkijalla olisi turvattu pitkäkestoinen taloudellinen riippumattomuus, niin hänellä olisi myös ajattelun ja uusien ideoiden kokeilun mahdollisuus. Toki se mahdollistaa myös lusmuilun, jos siihen on taipumusta. Eräs työhönsä kyllästynyt teologi sanoikin saatuaan pysyvän professuurin, että nyt tämä Saatanan tutkiminen loppui!

  6. Lentotaidoton sanoo:

    No niin, toivomus on että pikku hiljaa päästään tästä fantasioinnista ja sen arvioimisesta sekä ikiliikkujista itse asiaan, eli fysiikkaan. Kiitos.

    1. Syksy Räsänen sanoo:

      Blogin aiheena on fysiikan tutkimuksen sisällön lisäksi myös tutkimuksen tekeminen ja sen poliittinen ja sosiaalinen ympäristö.

      Eri aiheet kiinnostavat eri yleisöä, toisia (erityisesti muita tutkijoita) kiinnostaa tutkimusrahoituksen avaaminen, toisia pimeä aine.

      Seuraavan merkinnän aihe lienee pienet mustat aukot pimeän aineen ehdokkaana ja niiden tuottaminen Higgsin kentän avulla.

  7. Lentotaidoton sanoo:

    OK, hyvä näin. Sielunhoitoakin tietysti tarvitaan, mutta…

    ”Teksteissään hän ruotii pääasiassa kosmologian ja hiukkasfysiikkan aiheita”. Eli aiheita, joita otaksuisin useimpien tätä blokia lukevien haluavan käsiteltävän. Ei muuten, mutta kun ne ovat niin äärimmäisen kiinnostavia asiansatuntevan blogistin käsissä. Nykyään kun on kaikenlaista ”vaihtoehtoistakin totuutta” tarjolla.

    1. Syksy Räsänen sanoo:

      Sattumoisin tämä merkintä on herättänyt enemmän kommentteja ja kiinnostusta Twitterissä kuin mikään muu.

  8. Matias Slavov sanoo:

    Miten kilpailu saataisiin lakkautettua? Haluaisin pelata LA Kingsissa, mutta niin haluaa miljoona muutakin. Haluan tehdä akateemista tutkimusta, mutta niin haluaa kymmenen muutakin.

    1. Syksy Räsänen sanoo:

      En kirjoittanut kilpailun lopettamisesta, vaan kilpaillun rahoituksen vähentämisestä.

      Kilpailtu rahoitus viittaa rahoitukseen, josta päättävät ulkopuoliset tahot, kun taas perusrahoituksen kohdistamisesta päätetään yliopistoissa ja instituuteissa.

      Tekstissä mainitaankin tästä esimerkkejä: jatko-opiskelijoiden rahoituksen keskittäminen tohtoriohjelmiin sekä yllä siteeratun merkinnän malli, jossa jokaisen rahoituksensaajan pitää jakaa siitä tietty osa eteenpäin. Muitakin ehdotuksia on olemassa – ehkä palaan niihin tulevissa merkinnöissä.

      Toinen asia on sitten se, miten rahoituksesta kilpaillaan. Suuri ongelma on se, että järjestelmä kannustaa kirjoittamaan tutkimuksesta tavalla, joka ei vastaa todellisuutta, että haut ovat yhä raskaampia ja hakemusten arvioinnissa on paljon sattumanvaraisuutta eikä siinä kiinnitetä huomiota ennakkoluulojen tunnistamiseen ja kumoamiseen.

  9. Cargo sanoo:

    Sen sijaan, että tutkijat rukoilevat kädet kyynärpäitä myöden ristissä rahoitusta tuleville proggiksilleen, niin rahoitusta tulisi jakaa taannehtivasti menestyneille julkaisuille. Homman nimi olisi karkeasti se, että jos tuotos jää esijulkaisuasteelle, niin hakemus menee suoraan silppuriin. Näin maksimoidaan sekä ajan että rahoituksen käyttö.

    Ja jos ei kynä, paperi ja internet riitä, niin jokaisella taivaanrannan maalarilla on oikeus arvioida oman tutkimuksensa merkittävyyttä ja ottaa sen perusteella pankista lainaa haluamansa summan. Norjalaisilta saa nyt 50,000€ ilman vakuuksia.

    1. Syksy Räsänen sanoo:

      Tutkimuksen rahoittamisessa ei ole kyse vain yksilöiden hyödystä, vaan myös yhteisestä hyvästä.

  10. Luin twitter-linkin takana olevat kommentit. Siellä Tapio Ala-Nissilä kirjoitti ”Kun hakemusten hyväksymisprosentit putoavat 10% tai alle niin prosessi on satunnainen.” Olen samaa mieltä. Pohdiskelen tässä vähän taustalla olevia mekanismeja. Yksi juttu on että yksittäinen arvioija ei voi perehtyä niin moneen kuin kymmeneen hakemukseen. Tällöin käy niin että yksittäinen arvioija ei tyypillisesti näe yhtäkään esimerkkiä sellaisesta hakemuksesta joka menee läpi. Silloin arvioija ei tiedä absoluuttista skaalaa vaan helposti tulee gaussittaneeksi ne hakemukset jotka hän näkee. Tällöin esimerkiksi jos kaksi todella hyvää hakemusta sattuu samalle arvioijalle, hän ei anna molemmille yhtä paljon suitsutusta kuin ne ansaitsisivat.

    Ollaan siis pudottu tilanteeseen jossa hyväksymisprosentti on liian pieni. Kaksi välitöntä negatiivista vaikutusta ovat että arvointiprosessi on satunnainen jolloin rahoitus ei mene parhaille, ja lisäksi turha hakemusrumba vie hakijoiden ja arvioiden aikaa hukkaan. Kolmas negatiivinen vaikutus on että koska hakijat tietävät arvioinnin satunnaisluonteen, he täyttävät sen lottokupongin joka kerta, mikä ylläpitää hakemustulvaa ja matalaa hyväksymisprosenttia. Akatemian vastaus että lisäraha auttaisi toisi vain hetken helpotuksen.

    Sanoisin jos tietäisin mitä tilanteelle kannattaisi tehdä. Pitäisikö siirtyä sellaiseen että hakemukset ovat vain lyhyitä abstrakteja joita kollegat saavat äänestää, vähän kuin ehdokkaita eduskuntavaaleissa. Ja lisäksi laskettaisiin vanhoista meriiteistä jotkut jyvityspisteet jotenkin automaattisesti.

Vastaa

Sähköpostiosoitettasi ei julkaista. Pakolliset kentät on merkitty *

Ikiliikkujan pysäyttäminen

19.10.2018 klo 23.17, kirjoittaja
Kategoriat: Kosmokseen kirjoitettua

Ensi viikko on tieteen Open Access Week, ja keskiviikkona 24.10. on kello 12-14 Tiedekulmassa tutkimuksen avoimuuteen keskittyvä tilaisuus Kuka omistaa julkaisusi. Tilaisuuden Twitter-tunniste on #kukaomistaajulkaisusi. Minua pyydettiin mukaan, mutta koska olen Palestiinassa, osallistun vain lyhyellä etukäteen nauhoitetulla puheenvuorolla, joka esitetään paneelikeskustelun virikkeeksi. Olen aiemmin kirjoittanut aiheesta täällä, täällä, täällä, täällä, täällä ja täällä, ja muutaman vuoden takainen puheeni asian tiimoilta löytyy täältä.

Aihe on ajankohtainen, koska 4. syyskuuta julkistettiin kymmenen kohdan ohjelma Plan S open accessin toteuttamisesta Euroopassa. Open access tarkoittaa sitä, että tutkimus on kenen tahansa ilmaiseksi luettavissa. Suunnitelman takana on joukko Euroopan kansallisia tiedettä rahoittavia instituutioita, Euroopan komissio ja Euroopan tiedeneuvosto ERC. Suomen Akatemia, joka on (opetusministeriön jälkeen) Suomen tärkein tutkimuksen rahoittaja ilmoitti syyskuun 24. päivä tukevansa Plan S:n tavoitteita ja olevansa mukana edistämässä niitä. Myös nuoria tutkijoita edustavat Euroopan väitöskirjatutkijoiden neuvosto, Marie Curie -alumnien järjestö ja Young Academy of Europe ovat ilmaisseet tukensa.

Plan S:n ydin on se, että vuoden 2020 alkuun mennessä –alle kahden vuoden kuluttua– kaikki tutkimus, jota siihen osallistuvat tahot ovat rahoittaneet, pitää julkaista open accessina.

Nykyään tiedeyhteisö maksaa tieteellisille kustantajille ensin siitä, että tutkimus julkaistaan ja sitten siitä, että sen saa lukea. Open accessin tavoitteena on poistaa jälkimmäinen kustannuserä, ja Plan S on merkittävä askel kohti tätä. Suunnitelmassa on monia hyviä puolia, kuten se, että osallistuvat tahot luovat tarvittaessa kannustimia open access -julkaisujen perustamiselle ja rahoittavat niitä. Tärkeää on myös se, että rahoittajat seuraavat vaatimuksien noudattamista ja rankaisevat niiden rikkomisesta: ilman seurauksia Plan S uhkaisi jäädä vain yhdeksi julkilausumaksi muiden joukossa.

Harvardin yliopiston Open Access Project -hankkeen  johtaja Peter Suberilla on Plan S:stä tarkkanäköisiä kommentteja. Joukko eurooppalaisia tutkijoita on julkaissut suunnitelman vastaisen vetoomuksen, joka on sekin lukemisen arvoinen, vaikka en arvostelua täysin allekirjoitakaan.

Plan S:n vahvuus on se, että se on tieteentekijöiden ja heidän rahoittajiensa välinen sopimus. Open access -toiminnan yksi kompastuskivi on ollut kustantajakeskeisyys. Neuvotteluissa on yritetty vaikuttaa tieteellisiin kustantajiin, jotta nämä muuttaisivat lehtensä avoimesti luettaviksi ja laskisivat hintojaan. Tässä on kaksi ongelmaa.

Ensinnäkin kaupallisten kustantajien tavoitteena on voittojen maksimointi, joten niillä ei ole syytä laskea hintoja kuin pakon edessä. Niin kauan kuin tieteilijät antavat niille tutkimuksensa ilmaiseksi ja ostavat sen sitten takaisin, miksi muuttaa mitään? Tässä on hyvä historiikki tieteellisen kustantamisen kehittymisestä bisnekseksi, jota yksi sen perustajista kuvaili termillä ”perpetual financing machine”, jonka voi vapaasti suomentaa ”kassavirtojen ikiliikkujaksi”.

Toisekseen, tieteellisten kustantajien tarjoamat palvelut ovat vertaisarviointia lukuun ottamatta tarpeettomia, joten niiden hinnasta kiisteleminen on toissijaista. Mitä tulee julkaisujen avoimeen saatavuuteen, se on fysiikassa ja matematiikassa toteutettu jo yli 25 vuotta sitten: julkaisut laitetaan ilmaiseen nettiarkistoon arXiv, josta ne ovat kaikkien luettavissa.

Todellinen ongelma on vertaisarvioinnin järjestäminen kustannustehokkaalla tavalla. Tämäkin on ratkaistu: nk. overlay-lehdessä artikkelit julkaistaan vain ilmaisessa arkistossa kuten arXivissa, ja lehti hoitaa ainoastaan sen mihin sitä tarvitaan, eli vertaisarvioinnin. Tämä ei ole utopiaa, vaan toimiva julkaisumalli, jonka tunnetuin esimerkki on matematiikan lehti Discrete Analysis, jonka toimituskuntaan kuuluvat mm. Fields-mitalistit Tim Gowers ja Terence Tao. Omalla alallani on juuri aloittanut Open Journal of Astrophysics, ja muita esimerkkejä löytyy täältä.

Overlay-lehdet säästävät tieteellisen julkaisemisen kuluja tekijällä 400-500 verrattuna nykytilanteeseen ja tekijällä 100-200 kertaa verrattuna tilanteeseen, missä open access olisi kokonaan toteutunut. Mutta siinä missä arXiv kasvoi fysiikassa ja matematiikassa nopeasti tutkimuksen pääasialliseksi viestintäkanavaksi, overlay-lehdet ovat jääneet poikkeukseksi julkaisemisen kentällä.

Syynä on se, että koska tutkijat eivät maksa julkaisemisesta omasta budjetistaan, heillä ei ole syytä kiinnittää huomiota sen hintaan. Niinpä tieteilijät lähettävät artikkelinsa julkaistavaksi tuttuihin vanhoihin lehtiin, joilla on hyvä maine. Samasta syytä heitä ei kiinnosta käyttää aikaa overlay-lehtien perustamiseen.

Onkin valitettavaa, että Plan S:ssä mainitaan ilmaisten arkistojen merkitys ainoastaan arkistoimisen kannalta, ei julkaisemista ajatellen. Kun Plan S:ssä jo sitoudutaan uusien lehtien perustamiseen ja rahoittamiseen, seuraava askel olisi sen tekeminen tavalla, joka on nykyaikainen, kustannustehokas ja lähtee tiedeyhteisön, ei kaupallisten kustantajien, tarpeista.

Päivitys (05/11/18): Avoimen tieteen iltapäivän keskustelusta voi lukea täältä. Mukana on myös video keskustelusta, mukaan lukien lyhyt puheenvuoroni.

2 kommenttia “Ikiliikkujan pysäyttäminen”

Vastaa

Sähköpostiosoitettasi ei julkaista. Pakolliset kentät on merkitty *

Oppitunteja näkymättömästä maailmasta

6.10.2018 klo 13.57, kirjoittaja
Kategoriat: Kosmokseen kirjoitettua , Kosmologia

Kirjoitin Helsingin opettajien ammattiyhdistyksen lehden Rihveli numeroon 2/2018 artikkelin otsikolla Oppitunteja näkymättömästä maailmasta. Aiheena on pimeä aine esimerkkinä ymmärryksen hitaasta kasautumisesta fysiikassa. Viimeinen kappale on tämä:

Jos pimeän aineen tarinasta kirjoittaisi puhtaaksi vain lopulta oikeaksi osoittautuvan suunnan, tämä antaisi harhaanjohtavan kuvan siitä, miten tiede toimii. Ei ole viitoitettua polkua totuuteen, eikä yhtä tieteellistä metodia, joka kertoo miten edetä. Monet vihjeet osoittautuvat vesiperäksi, jotkut havainnot virheellisiksi ja useimmat teoreettiset ideat vääriksi. Tuntematonta kartoittaessa ei voi välttyä harharetkiltä.

6 kommenttia “Oppitunteja näkymättömästä maailmasta”

  1. Erkki Kolehmainen sanoo:

    Voisiko olla jopa niin, että pimeä aine on kuin flogiston? Siitä luopuminen olisi todellinen paradigman muutos, joka on tunnetusti vaikeaa kuten Thomas Kuhn on ansiokkaasti pohdiskellut.

  2. Aivan, mikään ei ole ennalta varmaa, ei edes se että jokin ongelma kuten pimeän aineen kysymys ylipäätään on ratkeava. Toki tiedekysymyksen ratkeavuus kannattaa ottaa työhypoteesiksi, koska päinvastainen työhypoteesi ei olisi hedelmällinen.

  3. 7v sanoo:

    käyttäytyykö kaikki materia
    dualistisesti (hiukkas,aalto)
    vai tunnetaanko
    mono tai trilistiä ilmiöitä
    ylipäätänsä?

    Entä mitenkä on asia
    pimeän aineen kanssa?

    ja onko tiedossa mitään
    hajautettua laskentaa menossa
    alkuteorialle
    josta johtuu kaikki 4 tunnettua
    perusvoimaa?

  4. Jernau Gurgeh sanoo:

    Moi.

    En oikein tiedä minkä merkinnän yhteyteen laittaisin tämän kysymyksen, joten menköön tähän. Ja tavallaan tässä onkin taka-ajatuksena se, että tarvitaanko pimeää ainetta selittämään esim. galaksien (uloimpien tähtien) rotaatiokäyriä. Kysymys on varmaan typerä, mutta kysyn kuitenkin, kun itseäni asia vaivaa.

    Kysymys koskee taitekerrointa ja siten gravitaatioaaltojen nopeutta: Onko gravitaatioaalloilla huomattu hidastumista (onko niillä sellaista ominaisuutta edes teoriassa) eri väliaineissa? Onko niiden nopeus aina c vai voiko se olla c/n? Onko mahdollinen n sama kuin valon tapauksessa (riippuen tietenkin väliaineesta) vai jotain ihan muuta?

    Jos taitekerroin on olemassa, mitä se aiheuttaa gravitaatiolle (avaruuden kaareutumiselle) vai aiheuttaako mitään?

    Konkreettinen esimerkki voisi kenties olla, että kaareutuuko avaruus tyhjässä avaruudessa samalla tavalla kuin vaikkapa maapallon tai tähden sisällä, jos alkuperäisen gravitaatioaallon aiheuttaja on samalla etäisyydellä? (Ja otetaan huomioon, että maapallo tai tähti aiheuttaa tietenkin itsessään myös gravitaatiota. En puhu siis tavallisesta monen kappaleen ongelmasta.)

    1. Syksy Räsänen sanoo:

      Kysymys on sen verta kaukana merkinnän aiheesta, että jää vastaamatta.

      Ursan lehden Tähdet ja avaruus vastauspalstalle voi muuten lähettää yleisiä tiedekysymyksiä.

Vastaa

Sähköpostiosoitettasi ei julkaista. Pakolliset kentät on merkitty *


Kissan kanssa laatikossa

29.9.2018 klo 21.06, kirjoittaja
Kategoriat: Kosmokseen kirjoitettua , Kosmologia

Olin viime viikolla Pariisissa APC-instituutissa konferenssissa The Universe as a Quantum Lab. Nimensä mukaisesti tapaaminen liikkui kvanttifysiikan ja kosmologian yhteisellä, enimmäkseen tuntemattomalla maaperällä.

Tällä hetkellä on kaksi perustavanlaatuista fysiikan teoriaa: yleinen suhteellisuusteoria ja kvanttikenttäteoria. Suhteellisuusteoria ymmärretään täysin, mutta kvanttifysiikasta on vielä käsittämättä yksi pieni seikka, nimittäin se, miksi maailma arkiskaalalla ei näytä sellaiselta kuin kvanttifysiikka kertoo.

Ei myöskään tiedetä sitä, miten nämä kaksi teoriaa sopivat yhteen, eli mikä on se perustavanlaatuisempi teoria, josta ne ovat approksimaatioita. Asian selvittämistä vaikeuttaa se, että kvanttifysiikalle ominaiset ilmiöt ovat yleensä merkittäviä vain pienessä mittakaavassa, kun taas yleinen suhteellisuusteoria tyypillisesti näyttäytyy vain isoilla etäisyyksillä.

Kosmologia on kuitenkin poikkeus. Varhaisessa maailmankaikkeudessa aineen tiheys on iso ja mittakaavat ovat pieniä, ja tilanteen kuvaamiseen tarvitaan sekä kvanttifysiikkaa että yleistä suhteellisuusteoriaa. On esitetty moneen suuntaan versovia ajatuksia siitä, miten nämä kaksi kohtaavat maailmankaikkeuden alkuhetkinä, esimerkiksi James Hartlen ja Stephen Hawkingin ehdotus, jossa maailmankaikkeus syntyy tyhjästä kvanttivärähtelynä. Toistaiseksi kuitenkin vain yksi idea on tuottanut ennustuksia, joita on onnistuneesti verrattu havaintoihin: kosminen inflaatio.

Inflaation aikana varhainen maailmankaikkeus laajenee kiihtyvällä tahdilla ja sen koko kasvaa ainakin tuhatmiljardismiljardiskertaiseksi. Tämän takia pienet kvanttivärähtelyt venyvät kosmisiin mittoihin ja jäätyvät. Nämä tyhjästä syntyneet epätasaisuudet ovat galaksien siemeniä, ja nykyään taivaalta voi lukea galaksien jakaumasta (sekä kosmisesta mikroaaltotaustasta), millaisia ensimmäisen sekunnin murto-osan kvanttivärähtelyt olivat.

Yksi APC:n konferenssissa käsitelty kysymys on se, kuinka tarkkoja jälkiä inflaation ajan tapahtumista on meille jäänyt. Inflaation aikainen valtava laajeneminen nimittäin pyyhkii epätasaisuuksien jakaumasta pois kvanttifysiikalle leimalliset ominaisuudet. Osaamme laskea, millaisia epätasaisuuksia kvanttivärähtelyt synnyttävät, ja havainnot vastaavat tuloksia. Mutta jokin klassisen fysiikan prosessi voisi periaatteessa synnyttää samanlaisen jakauman – sen kvanttikummallisuutta on vaikea nähdä. (Teknisesti sanottuna vaihtoehtojen välinen interferenssi on pieni.)

Viime aikoina on esitetty kiinnostavia ideoita, siitä miten inflaation kvanttiluonteeseen päästäisiin käsiksi, mutta toistaiseksi ei ole keksitty signaalia, joka olisi tarpeeksi iso, että sen erottaisi. Yritykset ovat kuitenkin osoittaneet sen, että inflaatiossa lähestytään sekä kysymystä kvanttifysiikan ja yleisen suhteellisuusteorian yhdistämisestä että sitä miksi arki ei näytä kvanttifysikaaliselta. Useimmiten nämä kysymykset pidetään erillään.

Kvanttifysiikan teoria kertoo todennäköisyydet eri havainnoille, mutta ei sitä, mikä niistä todella toteutuu. Yleensä ajatellaan, että tila määräytyy kun sitä havaitaan, eikä tarvita tätä yksinkertaista reseptiä enempää. Mutta jos haluaa tarkastella huolella, mitä havaitseminen tarkoittaa, joutuu pian ongelmiin.

Kosmologiassa ongelma tulee vastaan, koska havaitsija ja tutkittava systeemi eivät ole erillisiä, vaan me havaitsijat olemme osa havaitsemaamme systeemiä. Galaksien olemassaolo -ja siis meidän olemassaolomme- perustuu siihen, että nyt havaitsemillamme varhaisen maailmankaikkeuden kvanttivärähtelyillä on tietty arvo, joten tuntuu ylivoimaisen vaikealta ajatella että tämä arvo määräytyisi vasta sitä havaitessa. Kosmologiassa Schrödinger ja kissa ovat samassa laatikossa.

Kun aloin lukea kosmologien tutkimuksia aiheesta, minusta tuntui alkuun, että en oikein ymmärrä, miten kirjoittajat käsittävät tämän havaitsijan ja maailmankaikkeuden suhteen. Pariisin tapaamisessa hahmotin, että aiheen tutkijat eivät itsekään ole asiasta aivan perillä. Hämärä seutu perimmäisten kysymysten liepeillä kiehtoo, ja asiassa on päästy jonkin verran eteenpäinkin, dekoherenssin saralla.

Aina näiden ongelmien pohtimiseen ei ole suhtauduttu yhtä suopeasti kuin nykyään. 70-luvulla kvanttimekaniikan perusteiden tutkimuksella oli niin huono maine, että arvostettu julkaisu Physical Review antoi editoreilleen kirjallisen ohjeen, että jos aihetta koskevassa artikkelissa ei ole analyysiä uusista havainnoista, ainoastaan teoreettista tarkastelua, niin se hylätään suoralta kädeltä ilman vertaisarviointia. Niinpä alan pioneerin John Bellin tutkimusta julkaistiin kirjoituskoneella naputelluissa pienen levityksen lehtisissä.

Kvanttimekaniikan yhdistämisessä arkeen on kutkuttavaa se, että teorian matemaattinen rakenne on yksinkertainen ja selkeä, mutta havaintojen kuvaaminen siinä vaikuttaa tavattoman monimutkaiselta. Kvanttigravitaation tapauksessa itse teoriakin on oletettavasti vaikeaselkoinen.

APC:n konferenssissa näkyi miten paljon erilaisia ideoita kvanttigravitaatiosta ja kvanttifysiikasta vielä on. Osallistujilla oli erimielisyyksiä muun muassa siitä onko ollenkaan oikein käsitellä aika-avaruuden geometriaa kvanttifysikaalisesti ja siitä kuvaavatko kvanttimekaniikan todennäköisyydet uskomuksia, tietämättömyyttä vai todellisuuden ominaisuuksia? Toisaalta teorioiden ja kokeiden välillä on joskus yllättäviä kosketuspintoja. Esimerkiksi gravitaatioaaltojen havaitsemiseen tarvittavaa teknologiaa testannut satelliitti LISA Pathfinder teki tarkkoja mittauksia peilien vakaudesta avaruudessa, ja ne sattumoisin myös antavat tiukimman rajan tietyille malleille, joissa kappaleiden tila määräytyy koko ajan ilman mitään havaitsijaa.

Kokeiden eteneminen onkin vaikuttavaa. Nykyään laboratoriossa osataan valmistaa ja mitata interferenssi-ilmiöitä, joissa eri vaihtoehdot ovat 54 sentin päässä toisistaan. Schrödingerin kissan sijaan (joka on sekoituksessa elävää ja kuollutta) kokeessa atomi on sekoituksessa tilaa, jossa se valitsee oikealle tai vasemmalle menevän reitin. Schrödinger esitti ajatuskokeensa osoittaakseen, että kvantti-ilmiöiden ei pitäisi ulottua arkiskaaloille, mutta nyt vaihtoehtojen välinen etäisyys on jo kädenmitan verran, vaikka kappaleiden koko onkin kaukana kissoista. Saa nähdä löydetäänkö jotain kiinnostavaa ensin laboratoriossa vai taivaalta.

35 kommenttia “Kissan kanssa laatikossa”

  1. Eusa sanoo:

    Onko linkata aiheeseen tuosta Lisa Pathfinderin testauksessa löydetystä rajasta tilakoheraatioille?

    1. Syksy Räsänen sanoo:

      Angelo Bassin puhe konferenssissa:

      https://drive.google.com/file/d/1svrkH_-cO2hXIBvFCkEoliEynYHJP9AS/view

  2. Tapsa sanoo:

    Olin vuosi pari sitten Kari Enqvistin yleisöluennolla, jossa olin kuulevinani hänen sanovan, että heti inflaation jälkeen maailmankaikkeuden koko oli luokkaa yksi metri.

    Kuulinkohan oikein?

    1. Syksy Räsänen sanoo:

      Kysymys ei juuri liity merkinnän aiheeseen, joten en rupea vastaamaan siihen tässä.

      Mutta koska maailmankaikkeuden koosta toisinaan kysellään, saatan kirjoittaa siitä oman merkintänsä.

      1. Tapsa sanoo:

        Ok, asia vain palasi mieleeni tästä lausumastasi:

        ”Inflaation aikana varhainen maailmankaikkeus laajenee kiihtyvällä tahdilla ja sen koko kasvaa ainakin tuhatmiljardismiljardiskertaiseksi.”

        Tuostahan voi päätellä, että maailmankaikkeuden koko osataan arvioida sekä alkuhetkellä että inflaation päätyttyä – sillä tuskin lukua, vaikka iso onkin, sentään ihan hihasta on vetäisty.

        Mutta jään innolla odottamaan mahdollista palaamistasi asiaan!

        1. Syksy Räsänen sanoo:

          Tosiaan, tämä on tavallinen hämmennyksen aihe. Emme tiedä onko maailmankaikkeus äärellinen vai ääretön. Jos se on ääretön, se on aina ollut ääretön. Tällöin tuo ilmaisu viittaa siihen, että minkä tahansa maailmankaikkeuden äärellisen osan koko kasvaa tuolla tekijällä.

          Maailmankaikkeuden koosta vähän täällä:

          https://www.ursa.fi/blogi/kosmokseen-kirjoitettua/muotoja-ilman-mittanauhaa/

  3. Sunnuntaikosmologi sanoo:

    Onko sinulla mielipidettä siitä miten 70-luvun Physical Reviewin editorin, sellaisen jolla ei ollut tulevaisuudesta kertovaa kristallipalloa käytettävissä, olisi pitänyt menetellä ?
    En tunne kyseistä alaa, mutta voisin kuvitella että Bellin artikkeli oli yksi lukemattomien ihan hyvästä syystä unholaan jääneiden samaa aihetta käsittelevien artikkelien joukossa.

    1. Syksy Räsänen sanoo:

      En tunne tuon ajan tilannetta. Mutta kvanttimekaniikan perusteiden miettimisestä on sittemmin versonut tilan määräytymistä jne. tutkivien mallien lisäksi kvantti-informatiikka, joka on paljon tutkittu fysiikan haara.

      Toisaalta suurin osa fysiikan artikkeleista aiheesta kuin aiheesta jää hyvästä syystä unholaan, vain pienellä osalla on kestävää arvoa.

      1. Syksy Räsänen sanoo:

        On myös syytä ottaa huomioon, että julkaisut fysiikan lehdissä ovat tutkijoiden tärkein meriitti.

        Jos lehden johto päättää, että se ei hyväksy artikkeleita aiheesta X, niin he käyttävät merkittävää valtaa sen suhteen, mitä pitäisi tutkia ja mitä ei.

        (Tosin 1970-luvulla tilanne alan työpaikkojen suhteen taisi olla vähemmän kilpailtu ja vähemmän numeroihin keskittyvä kuin nykyään.)

  4. Jari Toivanen sanoo:

    Tuosta ”maailmankaikkeus syntyy tyhjästä kvanttivärähtelynä”. Onko se ymmärrettävä, että ensin oli klassinen laatikko tyhjää, jossa sitten tapahtui tyhjän kvanttivärähtely, vai voiko tyhjän kvanttivärähtelyä tapahtua ilman, että ensin on se tyhjä.

    1. Syksy Räsänen sanoo:

      Ajatuksena on se, että koko maailmankaikkeus, mukaan lukien aika ja avaruus, syntyvät tyhjästä. Pitää kuitenkin valmiiksi olla fysiikan lait, että näin voi tapahtua, ja tämä voi herättää kaikenlaisia filosofisia kysymyksiä.

      1. Jari Toivanen sanoo:

        Ja mistä ei voi puhua, siitä täytyy vaieta.

      2. Eusa sanoo:

        Ilmeisestikin eräs filosofia on, että koska ”matematiikka on ikuista”, luonnonlait perustuisivat pohjimmiltaan matematiikkaan – esim. lukuteoriaan, jolla toisiinsa muuten sekoittuvat määrät ulottuvuuksissa voidaan jäsentää mm. alkulukujaollisuuksin.

        Tuollaisen fundamentaalimatematiikan tutkiminen vailtettavan usein johtaa numerologisiin virityksiin. On kuitenkin joitain ihan varteenotettavia matemaattisia ilmiöitä, joilla on helposti todettavia kytköksiä siihen mitä fysikaalisesta todellisuudesta havaitsemme – esim. Riemannin zeetta-funktion ominaisuudet ovat mielestäni sellaista kauraa.

        Onko sinun kohdallesi osunut jotain matematiikkaa, jossa olisit nähnyt ihan fundamentaalia fysiikan lainalaisuuden mahdollisuutta? (ei vain mitatun ilmiön kuvausmatematiikkana).

        1. Syksy Räsänen sanoo:

          Kysymys ei juuri liity merkinnän aiheeseen, joten ei siitä sen enempää.

          1. Eusa sanoo:

            No, merkitse muistiin vaikka pyyntönä omalle merkinnälleen ”matematiikan rooli fyysisen todellisuuden mahdollistajana”.

  5. Jorma Kilpi sanoo:

    Olen aina ihmetellyt miten kissa voitaisiin saada sellaiseen tilaan että se ei vuorovaikuta muun maailmankaikkeuden kanssa. Olen siinä (harha?)käsityksessa, että vuorovaikutuksessa informaatiota siirtyy ja informaation siirtyminen on mittaus riippumatta siitä tulkitseeko joku sitä informaatiota. Kvanttitasolla erilliset objektit voivat jotenkin kummasti jakaa yhteisen informaation sopivan vuorovaikutuksen jälkeen. Tai ovatko ne vuorovaikutuksen jälkeen enää erillisiä objekteja?

    1. Syksy Räsänen sanoo:

      En ole varma ymmärränkö kysymystä.

      Se, mitä ”mittaus” tarkoittaa on kvanttimekaniikassa monimutkaista.

      Mutta kissan ja havaitsijan tilan kytkeytymisessä toisiinsa (teknisesti sanoen lomittumisessa) on tosiaan kyse informaation välittämisestä.

      Asiaan liittyvästä dekoherenssista (joka ei ratkaise ongelmaa mittauksesta) hieman täällä:

      https://www.tiede.fi/blogit/maailmankaikkeutta_etsimassa/maarattyina_yhteen

      1. Eusa sanoo:

        Eikös ongelma mittauksesta ole turha? perustuuko se ajatukselle, että olisi kiva jos mittaus olisi erikseen, eikä mitenkään häiritsisi mittauskohdetta?

        No, onhan itsestään selvää, että kaikki mittalaitteet ovat osa sitä todellisuutta, jota niillä mitatataan. Eikö mittaus ja vuorovaikutus voida täysin rinnastaa, jolloin todellisuus sisältää vain hyvin paljon mittauksia, joista emme vajavaisella tekniikalla vain saa mitään informaatiota? Ja täydellinen determinismi on mahdollinen skenaario…

        1. Lentotaidoton sanoo:

          Eusa.
          Itse olen käsittänyt että mittauksen ”ongelma” ei ole tekninen (tämähän oli se alkuperäinen, varhainen ”selitys” epämääräisyydelle). Vaan sisäänrakennettu teoriaan. Tietysti ”sisäänrakennettu” ei ole aina samaa kuin selitys. Joka tapauksessa täydellinen determinismi edellyttäisi mielestäni ei-lokaalisuutta. Lomittumisessa ei tapahdu informaationvaihtoa (yksi ja sama kvanttitila). Informaationvaihto vasta dekoherenssissä. ”Ongelma” on mittaus/mittaaja ja dekoherenssi, jotka pitäisi voida ”poisselittää”.

          Vakioheitto tähän ongelmaan ovat piilomuuttujat ja monimaailmatulkinta, jotka eivät oikein ”kolahda” (voi hyvin johtua rajoittuneisuudestani).

  6. Kari O. sanoo:

    Tämäkään kysymys ei liity aiheeseen, mutta kysyn silti, koska se joskus aiemmin (aiheeseen liittyvänäkin) on jäänyt vastaamatta:
    – Usein kirjoitetaan mm. Tähdet ja Avaruus lehdessä, että mustan aukon sisältä ei ”mikään pääse karkaamaan ulos”, (jos ei oteta lukuun Hawkingin säteilyä). Miten on mahdollista että ”näemme” sen sisällä olevan massan aikaansaaman gravitaation?

    Voidaan tietenkin sanoa, että kyse ei ole minkään ulos karkaamisesta, kun pohjimmiltaan siinä gravitaatiossa on kyse avaruuden kaareutumisesta, mutta jotenkin tämä ei vielä riitä vastaukseksi.

    1. Syksy Räsänen sanoo:

      Blogin kommenttiosio on tarkoitettu kysymyksille, kommenteille ja keskustelulle, joka liittyy kuhunkin merkintään. Toiveet tulevien merkintöjen aiheista ovat myös tervetulleita.

      Tämä ei ole paikka yleisille kysymyksille mistä tahansa aiheesta fysiikassa.

      1. Kari O. sanoo:

        Ensimmäisellä kysymyskerralla (vuosia sitten) kysymys liittyi kirjoituksen aiheeseen. Varmaan ennemmin tai myöhemmin tulee uusi blogi joka osuu paremmin kuin tämä. Vaan liekö juuri tuota kysymystä kukaan muu aiemmin esittänytkään.

  7. Kari O. sanoo:

    Jos kaikki maailmankaikkeuden ”stuff” on joskus ollut yhdessä pisteessä, eikö tästä seuraa, että kaikki ”stuff” mitä maailmankaikkeudessa on, on kvanttilomittunut keskenään?

    Tämähän periaatteessa mahdollistaa jonkin lajin kaukovaikutuksen melkein minkä tahansa ”otusten” kesken missä tahansa.

    (Lainaan hieman Raimo Keskisen tyyliä, koska se tässä aiheessa näyttää toimivan, jos ei halua erityisesti nimetä jotakin hituja tms.)

    1. Syksy Räsänen sanoo:

      Kaikki maailmankaikkeuden aine ei ole ollut yhdessä pisteessä.

      Pitää kuitenkin paikkansa, että näkemämme maailmankaikkeuden kaikki osat ovat olleet kosketuksissa hyvin varhaisina aikoina.

      En ole varma mitä tarkoitat tässä kaukovaikutuksella. Kvanttimekaniikan lomittumiseen ei liity kaukovaikutusta siinä mielessä, että informaatiota välittyisi valoa nopeammin.

      1. Lentotaidoton sanoo:

        Räsänen: Pitää kuitenkin paikkansa, että näkemämme maailmankaikkeuden kaikki osat ovat olleet kosketuksissa hyvin varhaisina aikoina.

        Eli muutenhan ei (havaittava) kosmoksemme olisi näin flat (elikä oomega lähes nolla) sekä näin isotrooppinen ja homogeeninen. Selittyy inflaatiolla, joka ”eristi” havaittavan kosmoksemme kausaalisesti yhtenäiseksi.

        1. Syksy Räsänen sanoo:

          Joo, tosin laakeus ja kausaalinen yhteys ovat eri asioita, vaikka seuraavatkin molemmat inflaatiosta. (Notaatio on muuten sellainen, että laakean maailmankaikkeuden Omega=1.)

      2. Kari O. sanoo:

        Kaukovaikutus tulkinnasta riippuen tarkoittanee joko korrelaatiota tai kausaliteettia kaukaisten ilmiöiden välillä siten että tapahtumien välinen aika on lyhempi kuin niiden välinen etäisyys valo(aika)yksiköissä. (Eli ei-lokaalisuus?)

        Pitäydytään korrelaatiossa. Periaatteessa se mahdollistaisi ”oudolla tavalla” samanaikaisia ilmiöitä maailmankaikkeuden eri puolilla. Mitä ne voisivat olla?

  8. Lentotaidoton sanoo:

    ”Notaatio on muuten sellainen, että laakean maailmankaikkeuden Omega=1.”

    Joo sori, kirjoitusvirhe. Siis juuri kuten inflaatiomallit ennustavat.

  9. Jernau Gurgeh sanoo:

    Syksy, totesit:

    ”Ajatuksena on se, että koko maailmankaikkeus, mukaan lukien aika ja avaruus, syntyvät tyhjästä. Pitää kuitenkin valmiiksi olla fysiikan lait, että näin voi tapahtua, ja tämä voi herättää kaikenlaisia filosofisia kysymyksiä.”

    Tähän liittyen Lee Smolinilla on ajatus ajan todellisuudesta: ”Time is real”.

    En ole siihen perusteellisesti tutustunut, mutta hänen pääargumenttinsa taitaa olla, että ei voi olla olemassa luonnonlakeja ajan ulkopuolella. Toisin sanoen ”luonnonlait” kokevat ajan ja siten ne muuttuvat evoluution myötä niin kuin kaikki muukin maailmankaikkeudessa. Ja sitä miten ne muuttuvat, ei voi ennustaa, koska muuttumisen ennustaminen tarvitsisi taakseen ajassa muuttumattoman luonnonlain, ja niitähän ei Smolinin argumentin mukaan juuri ole.

    Oletko sinä perehtynyt yhtään Smolinin argumentteihin, onko niissä mitään ideaa?

    Nuo filosofiset ongelmat, jotka mainitsit, koskettaa myös matematiikkaa. Onko numerot eli matematiikka olemassa?

    Platonismin mukaan se on olemassa abstraktina ajasta ja avaruudesta riippumattomana oliona. Näin ollen matematiikka on luonnonlait.

    Nominalismin mukaan se on konkreettista ajassa ja avaruudessa. Se on siis syntynyt yhdessä maailmankaikkeuden ja luonnonlakien kanssa ja on tarkka kuvaus maailmankaikkeudesta.

    Fiktionalismin mukaan se on vain ihmisen keksimä työkalu, jolla ei ole mitään yhteyttä mihinkään. Sillä voidaan vain tehdä eri tarkkuudella toimivia approksimaatioita maailmankaikkeudesta.

    Onko sinulla fyysikkona tähän jotain kantaa tai ajatusta asiasta, kuinka itse koet tämän?

    1. Syksy Räsänen sanoo:

      En osaa sanoa, ovatko esimerkiksi kokonaisluvut olemassa. Jos ovat, en tiedä mitä olemassaolo tarkoittaa, mutta kummalta tuntuu myös sanoa, että ne eivät ole.

      Ajatuksiani asiasta vähän täällä:

      https://www.tiede.fi/blogit/maailmankaikkeutta_etsimassa/kaikki_jarjestyksessa

    2. Kari O. sanoo:

      Kieli ainakin jossain määrin ohjaa ajatteluamme tässä.

      Onko niin että luonnonlait ’löydetään’ tai ’uudelleen-löydetään (re-search)’ vai onko niin että ne ’keksitään’.

      Edellinen vaihtoehto viittaa siihen, että löydetään jotain joka on (aina ollut) olemassa, tässä maailmankaikkeudessa ainakin. Jonkinlainen Platonin idea siis.

      Jälkimmäinen siihen että ihminen keksii täysin uutena ideana jotain jota ei luonnossa ole (koskaan ollut). Artefakti siis. Tuskinpa kukaan löytää luonnosta esim. 10 miljardin transistorin mikrosiruja, vaikka ne perustuvat tunnettuihin luonnonlakeihin.

      Pohjimmiltaan kysymys lienee jotain sellaista, että voiko ihminen keksiä sellaisen uuden luonnonlain, jota luonnossa ei vielä ole ollut.. mikä ei vaikuta kovin todennäköiseltä. Se voisi silti olla jopa ihan toimiva työhypoteesi sille, joka vielä haluaa jotain uusia Nobelin arvoisia luonnonlakeja löytää. 😉

      Toki voidaan löytää esim. sellaisia uusia kvantti-ilmiöitä, joita ei vielä hetkeä aiemmin osattu mitenkään kuvitella. Hyvänä esimerkkinä Aalto-yliopiston viimeaikaiset tutkimustulokset kvanttiteknologiassa.

  10. ”Kosmologiassa Schrödinger ja kissa ovat samassa laatikossa.” Tämä on hyvä pointti. Itse tykkään ajatella karkeasti ottaen näin: Koska tosiaan kosmologiassa ne ovat samassa laatikossa, saman pitäisi yhdenmukaisuussyistä päteä normaaliinkin Schrödingerin kissaan, ja siksi monimaailmatulkinta. Tai siis jokin tulkinta jossa havaitsijakin haarautuu.

Vastaa

Sähköpostiosoitettasi ei julkaista. Pakolliset kentät on merkitty *


Kaivon äärellä

19.9.2018 klo 01.10, kirjoittaja
Kategoriat: Kosmokseen kirjoitettua , Kosmologia

Vietän syyslukukauden Birzeitin yliopistossa, Länsirannalla, Miehitetyillä palestiinalaisalueilla kosmologiaa luennoiden. Vierailin Birzeitissä ensimmäisen kerran neljä vuotta sitten.

Birzeitin kampusaluetta.

Birzeit oli ensimmäinen palestiinalainen yliopisto ja se on vieläkin arvostetuin. Vuonna 1924 Birzeitin (nimi tarkoittaa oliiviöljykaivoa) kylään perustetusta koulusta se kasvoi vuonna 1975 yliopistoksi. Instituutio on kokenut kovia miehityksen alla. Miehityshallinto on useita kertoja pakolla sulkenut yliopiston – pisin sulku kesti 1980-luvulla 4 vuotta ja 3 kuukautta. Henkilökunta järjesti tällöin luentoja salaa, minkä Israel tuomitsi ”laittomina opetussoluina”, joista määrättiin 10 vuoden vankeusrangaistus.

Nykyään armeija ei puutu yhtä jyrkästi yliopiston toimintaan, mutta Länsirannalle rakennettu erottelumuuri, tarkastuspisteet, suljetut tiet ja muut fyysiset ja byrokraattiset kulun esteet rampauttavat yliopistoja kuten muutakin yhteiskuntaa. Lisäksi miehitysjoukot hyökkäävät kampukselle ja kaappaavat opiskelijoita, erityisesti ylioppilaskunnan hallituksen jäseniä. Tämä miehityksen tunkeutuminen yliopistolle oli yksi inspiraatio yliopistokampukselle sijoittuvalle palestiinalais-suomalaiselle larpille Piiritystila, joka siirsi Palestiinan tilanteen Suomeen. (Minulla oli ilo olla mukana konsulttina ja avustajana; minun ja muiden kokemuksia on kerätty dokumentaatiokirjaan.)

Israel on myös viime aikoina estänyt yhä useammalta ulkomaiselta luennoitsijalta pääsyn Länsirannalle ja alkanut evätä jatkoviisumeita niiltä, jotka jo Birzeitissä. Järjestö Scientists for Palestine yrittää osaltaan estää palestiinalaisten yliopistojen eristämistä järjestämällä konferensseja ja muita yhteyksiä Palestiinan ja muiden maiden opiskelijoiden ja tiedeyhteisön välille.

Näissä olosuhteissa Birzeitin yliopisto on kasvanut ja kehittynyt. Nykyään se on kooltaan noin kaksi viidennestä Helsingin yliopistosta: opiskelijoita on noin 14 000, joista yli puolet on naisia. Fysiikan opinnot aloittaa vuosittain noin 30-40 opiskelijaa. QS World University Rankingin mukaan (mitä tällaiset arviot sitten mittaavatkaan) Birzeitin yliopisto kuuluu maailman yliopistojen 3% kärkeen. Tarkemmin yliopiston vaiheista voi lukea sen pitkäaikaisen johtajan Gabi Baramkin omakohtaisesta kirjasta Peaceful Resistance: Building a Palestinian University Under Occupation.

Matemaattis-luonnontieteellinen tiedekunta.

Vanhemman polven tutkijoista monet ovat väitelleet Neuvostoliitossa, nuoremmista Yhdysvalloissa ja Euroopassa, tutkien esimerkiksi mustia aukkoja säieteoriassa. Birzeitissa tutkimukselle ei kuitenkaan jää juuri aikaa, melkein kaikki päivät menevät opetukseen ja hallintoon. (Kuulemma fyysikoiden katsotaan soveltuvan hallintohommiin erityisen hyvin ja heillä on niitä paljon.) Henkilökunnan jäsenillä on mahdollisuus saada välillä lyhyitä vapaita kausia tutkimusta varten, ja Birzeitin fyysikot vierailevat ulkomaisissa yliopistoissa ja tutkimuslaitoksissa, mutta tämä ei korvaa sitä, että tutkimustyöhön ei voi keskittyä pitkäjänteisesti ja jatkuvasti.

Ongelmana on raha. Miehityksen alla ja sen osana toimiva ja ulkoisesta avusta riippuvainen Palestiinalaishallinto ei rahoita yliopistoja (eikä etenkään tutkimusta) riittävästi. Suurin osa käyttöbudjetista haalitaan lukukausimaksuista; lisäksi on lahjoituksia, joita annetaan erityisesti rakennuksia varten. Koska yliopistolla on talousvaikeuksia, järjestelmä johtaa siihen, että opiskelijoita otetaan sisään löyhin kriteerein budjetin kartuttamiseksi.

Se, että opiskelijoiden valintaperusteisiin vaikuttaa yliopiston rahantarve opiskelijoiden tason kustannuksella ei palvele sen enempää opiskelijoita kuin yhteiskuntaa, vaikka Suomeenkin lukukausimaksuja jotkut ajavat. Järjestelyssä on toki se hyvä puoli, että yliopisto on hallinnosta riippumaton, toisin kuin Suomessa, missä nimellisesti autonomisen Helsingin yliopiston johto seuraa uskollisesti opetusministeriön ohjeita.

Talousvaikeudet näkyvät myös kosmologiakurssin arjessa. Monet opiskelijat ovat töissä rahoittaakseen opintonsa, eivätkä pääse kampukselle joka päivä. Kurssin luennot ovat siksi kaikki samana päivänä, kolme tuntia peräjälkeen, vaikka keskittymisen ja hahmottamisen kannalta ne kannattaisi jakaa eri päiville. Kaikille sopivan laskuharjoitusajan löytäminen on samasta syystä vaikeaa. Onneksi kurssilaisten motivaatio on korkea, ja luennoilla on enemmän kysymyksiä ja vuorovaikutusta kuin Suomessa.

Maisteriopiskelijat saavat stipendejä, mutta kaikki eivät ota niitä vastaan, tai ottavat vain puoliksi ja jatkavat töitä hieman pienemmällä tuntimäärällä. Länsirannan nuorisotyöttömyys on 40%, joten työpaikoista halutaan pitää kiinni. Valmistuminen yliopistosta ei valitettavasti sekään takaa työllistymistä, ja monet päätyvät töihin, joilla ei ole mitään tekemistä opintojen kanssa.

Koulutuksella on kuitenkin kova kysyntä, koska sitä arvostetaan yhteiskunnassa. Palestiinalaisten koulutusaste on korkea, ja pitkään monet Persianlahden maiden lääkärit ja insinöörit olivat kotoisin Miehitetyiltä palestiinalaisalueilta. Vieläkin on se ongelma, että monet parhaista valmistuneista lähtevät ulkomaille, kun apartheid-järjestelmä estää talouden kehittymisen ja polkee poliittisia ja henkilökohtaisia vapauksia. Toisaalta yliopistojen kehittämä laajempi näkökulma, kriittinen tarkastelu ja ajattelun varmuus on omiaan auttamaan vapauden tavoittelussa, ja kuten Suomessa, myös Palestiinassa yliopistoilla on tärkeä rooli kansakunnan kehityksessä.

Kampusalueella sijaitseva Palestiinan museo.

13 kommenttia “Kaivon äärellä”

  1. Sunnuntaikosmologi sanoo:

    Kun olosuhteet ovat noin rankkoja, ja prioriteetit varmaankin ihan muualla, niin onko palestiinalaisten omissa yliopistoissa luonnontieteiden kohdalla mitään kansainvälistä tasoa olevia tutkimusryhmiä ?

    1. Syksy Räsänen sanoo:

      Ei ainakaan hiukkasfysiikassa ja kosmologiassa, muista aloista en tiedä. Jotkut tutkijat ovat mukana ulkomaisissa tutkimusryhmissä tekemässä korkealaatuista tutkimusta.

  2. Lentotaidoton sanoo:

    No kuinka tämän blogin käy? Onko kaivo liian syvä ja kuiva?

    1. Syksy Räsänen sanoo:

      Blogi jatkuu, kuten näkyy. Seuraavaksi on vuorossa havaintoja kosmologian ja kvanttifysiikan risteyksestä konferenssista, jossa tällä viikolla olen Pariisissa.

  3. Erkki Kolehmainen sanoo:

    Minua ihmetyttää suuresti tuo Syksy Räsäsen mainitsema yliopiston rahoituksen puute? Vaikka Irak, Syyria ja Libya ovat USA:n ”demokratiahankkeiden” vuoksi tilapäisesti maksukyvyttömiä, niin onhan niitä vielä jäljellä rikkaita arabimaita. Vai onkohan osasyynä se ettei niissä ehkä arvosteta tiedettä? Kun Saksaan tuli valtava pakolaisvyöry, niin Saudi-Arabia tarjoutui kustantamaan heille 200 uutta moskeijaa. Olen sitä mieltä, että osa palestiinalaisten ongelmista ovat itseaiheutettuja! Opetetaanko Birzeitin yliopistossa shariaa?

    1. Syksy Räsänen sanoo:

      Palestiinalaishallinnon suurin rahoittaja on EU ja EU-maat, ei arabimaat.

      Palestiinalaishallinnolla ei ole mahdollisuuksia taloudelliseen riippumattomuuteen, koska se toimii sotilasmiehitystilanteessa. Palestiinalaishallinnon suurin kuluerä on turvallisuusjoukot, jotka osallistuvat miehityksen ylläpitämiseen Israelin miehitysjoukkojen kanssa.

      Toisin kuin Helsingin yliopistossa, Birzeitin yliopistossa ei ole teologista tiedekuntaa. Laitoksista ja opinto-ohjelmista on tietoa täällä:

      https://www.birzeit.edu/en/study/academic-departments

  4. mikko kalevi siitonen sanoo:

    Tuota noin, olisiko se ihan noin simppeli juttu kuin kirjoitat. Voihan olla etta Israelin hallitus lyo, mutta silti joskus tuntuu, etta jos nilla kuitenkin on syynsa moiseen apartheidiin?

    Ei se ihan mukava historia ole, mita olen nahnyt. Ehka olen propagandan uhri, valaise minua. Tuli vaan mieleen. Muuten olen taman plogin innokas lukija, aina kiinnostaa.

    Olenko oikessa vai vaarassa. Minusta Israelilla on syynsa moiseen?

    t. Mikko

    1. Syksy Räsänen sanoo:

      YK:n vuoden 1973 apartheid-yleissopimuksen mukaan apartheid on kansainvälisen oikeuden rikkomus.

      Geneven neljännen sopimuksen vuoden 1977 ensimmäisen lisäpöytäkirjan mukaan apartheid on sotarikos.

      Kansainvälisen rikostuomioistuimen ICC vuoden 2002 perussäännön mukaan apartheid on rikos ihmisyyttä vastaan.

      On uskomatonta, että näin vakaville rikoksille vielä löytyy puolustajia.

      Kansainvälisestä oikeudesta ja historiasta voi lukea tarkemmin kirjastani ”Israelin apartheid”:

      https://intokustannus.fi/kirja/israelin-apartheid/

      1. mikko kalevi siitonen sanoo:

        Noin kai se sitten on.

        t. Mikko

        1. Erkki Kolehmainen sanoo:

          Syksy Räsänen voisi tehdä vastaavan listan siitä, mitä kansainväliset sopimukset sanovat terrorismista. Israelin rakentama muuri on käytännössä estänyt itsemurhaiskut juutalaisia vastaan Israelissa. Se on hyvä asia olkoonkin vaikka sitten apartheidia.

          1. Syksy Räsänen sanoo:

            Israelin turvallisuuspalvelu Shin Betin mukaan pääasiallinen syy itsemurhaiskujen vähenemiseen ei ollut muuri, koska sen ohittaminen on helppoa. Israelissa käy laittomasti 30 000-60 000 palestiinalaista töissä muurin yli, ali, ohi tai läpi: hyökkääjien olisi helppo käyttää samoja reittejä.

            Kansainvälinen tuomioistuin (ICJ) YK:n korkein oikeudellinen elin, on todennut, että muuria ei voi oikeuttaa turvallisuusargumenteilla. Yksi syy tähän on se, että se ei ole Israelin rajalla, vaan syvällä Miehitettyjen palestiinalaisalueiden sisällä, eli se laittomasti liittää alueita Israeliin. ICJ:n mukaan muuri on laiton, ja kaikkien Geneven neljännen sopimuksen ratifioineiden valtioiden (eli kaikkien maailman maiden) pitää toimia sen purkamiseksi.

            ICJ:n raportin voi lukea täältä: https://www.icj-cij.org/en/case/131

            Lisää muurista voi lukea kirjani Israelin apartheid luvusta 8: https://intokustannus.fi/kirja/israelin-apartheid/

            Tässä blogissa ei enää julkaista kommentteja, joissa puolustetaan sotarikoksia ja rikoksia ihmisyyttä vastaan. Sellaisia kommentteja lähettävät saavat pysyvän kommentointikiellon.

Vastaa

Sähköpostiosoitettasi ei julkaista. Pakolliset kentät on merkitty *


Pimeä sydän

30.8.2018 klo 12.58, kirjoittaja
Kategoriat: Kosmokseen kirjoitettua , Kosmologia

Vaikka pimeä aine on paras selitys monille tähtitieteen ja kosmologian havainnoille, varmuuden saisi vain löytämällä pimeän aineen hiukkasen. Tähän on perinteisesti ajateltu olevan kolme tapaa: suora havainto, epäsuora havainto ja kiihdytinhavainto. Suora havainto tarkoittaa sitä, että hiukkanen havaitaan laboratoriossa, epäsuora sitä, että hiukkasen hajoamis- tai annihilaatiotuotteet nähdään taivaalla ja kiihdytinhavainto sitä, että hiukkanen tuotetaan hiukkaskiihdyttimissä.

Neljänneksi mahdollisuudeksi on viime vuosina noussut astrofysikaalinen havainto (astrofysiikka on hienompi tapa sanoa tähtitiede), eli sen havaitseminen, miten pimeä aine vaikuttaa tähtitieteen ilmiöihin. Eräs kiinnostava spekulatiivinen idea tältä saralta on pimeät tähdet. Heti alkuun on syytä kiiruhtaa sanomaan, että tämä nimi (jolla on historiaa elokuvan saralla) on harhaanjohtava. Mustat aukot todella ovat mustia, mutta pimeät tähdet ovat kirkkaita: itse asiassa niille on leimallistase, että ne ovat paljon kirkkaampia kuin tavalliset tähdet. Nimi tarkoittaa vain sitä, että tähden tärkein energianlähde on pimeän aineen annihilaatio.

Useissa pimeän aineen malleissa (kuten nynnyjen tapauksessa) pimeä aine koostuu sekä hiukkasista että antihiukkasista. Kun hiukkanen ja antihiukkanen kohtaavat, ne annihiloituvat kevyemmiksi hiukkasiksi, tyypillisesti enimmäkseen fotoneiksi ja neutriinoiksi. Koska maailmankaikkeus laajenee, aineen tiheys laskee ajan kuluessa – eli aine on ennen ollut tiheämpään pakattua. Niinpä varhaisessa maailmankaikkeudessa pimeä aine ja antiaine annihiloituivat esteettä keskenään, mutta tiheyden laskiessa hiukkaset eivät enää löydä toisiaan ja ainetta ja antiainetta jää jäljelle.

Mutta samalla kun maailmankaikkeuden keskitiheys laskee, syntyy tihentymiä kuten galakseja gravitaation kasatessa ainetta. Kun maailmankaikkeus on noin 200 miljoonaa vuoden ikäinen, joissakin harvoissa paikoissa aineen tiheys kasvaa niin isoksi, että ydinreaktiot käynnistyvät: toisin sanoen, tähdet syttyvät. Tavallisten tähtien energianlähde on keskustan fuusioreaktiot. Kun ydinpolttoaine loppuu, tähti romahtaa, räjähtää ja syytää reaktioissa syntyneet alkuaineet pölyksi, josta uudet tähdet ja planeetat rakentuvat.

Koska pimeä aine ei pysty jäähtymään, se ei rusahda kasaan yhtä tehokkaasti kuin tavallinen aine. Tämän takia pienistä klimpeistä, kuten tähdistä ja planeetoista, suurin osa on näkyvää ainetta. Mutta vaikka tähden massasta alle tuhannesosa olisi pimeää ainetta, se voi olla vastuussa suurimmasta osasta tähden energiantuotannosta, koska annihilaatio on erittäin tehokasta. Hiukkasen ja antihiukkasen hävittäessä toisensa kaikki niiden massaa vastaava energia muuttuu hiukkasiksi ja tähden sisällä lämmöksi. Ydinreaktioissa vapautuu vain noin prosentti ydinten energiasta.

Nykyisissä tähdissä pimeällä aineella ei ole havaittavaa roolia, mutta varhaisina aikoina pimeän aineen tiheys oli isompi, joten annihilaatiota oli enemmän. Pimeät tähdet ovat voineet olla tähtien kehityksen ensimmäinen vaihe.

Pimeät tähdet ovat isoja, niiden säde on yli tuhat kertaa Aurinkoa isompi, mutta annihilaatio on tiukasti rajoittunut keskustaan, ja tähtien pinta on melko kylmä, 10 000 astetta. Tavallisten tähtien kasvua rajoittaa se, että niiden kuumasta pinnasta nouseva aurinkotuuli piiskaa ainetta poispäin. Pimeillä tähdillä ei ole tätä ongelmaa, joten ne voivat ahmia kaasua ja pimeää ainetta ja lihoa yhdestä Auringon massasta kymmenen miljoonaa kertaa Aurinkoa raskaammiksi jäteiksi.

Lopulta, miljoonien tai miljardien vuosien jälkeen, tähden pimeä aine loppuu ja tähti romahtaa. Siitä voi syntyä musta aukko. Pimeitä tähtiä onkin ehdotettu ratkaisuksi siihen, että varhaisessa maailmankaikkeudessa näkyy miljardien Auringon massaisia mustia aukkoja. On vaikeaa ymmärtää, miten tavallisista tähdistä ehtii muutamassa sadassa miljoonassa nopeasti niin valtavia mustia aukkoja, mutta pimeiltä tähdiltä se voisi onnistua.

Tämä on kiinnostava selitys, mutta pimeiden tähtien idea tekee myös ennusteita. Ne ovat hyvin kirkkaita, jopa kymmenen miljardia kertaa Aurinkoa kirkkaampia. Vuonna 2021 kiertoradalle nouseva James Webb Space Telescope voisi havaita yksittäisiä pimeitä tähtiä varhaisessa maailmankaikkeudessa.

Muillakin kappaleilla saattaa olla pimeän aineen sydän. Maa kaappaa gravitaatiollaan pimeän aineen hiukkasia Aurinkokunnan kiertäessa ympäri Linnunrataa, ja jos hiukkaset vähän törmäilevät toisiinsa ja Maan atomeihin (kuten nynnyt ja vastaavat hiukkaset tekevät), ne menettävät hiljalleen energiaa ja päätyvät lopulta gravitaatiokuopan pohjalle, Maan ytimeen. Maan gravitaatio on kuitenkin niin heikko, että annihilaatio ei ole kovin voimakasta, ja sen havaitseminen on vaikeaa, etenkin kun vain neutriinot pääsevät maan alta pakoon. Näitä jalkojemme alla tapahtuvan annihilaation tuotteita on etsitty, mutta mitään ei ole löytynyt.

Eräs ajankohtainen idea on pimeän aineen löytäminen gravitaatioaaltojen avulla. Suoraviivaisinta olisi havaita pimeiden tähtien yhtymisestä syntyviä gravitaatioaaltoja. Siitä syntyy erilaisia gravitaatioaaltoja kuin tähtien tai mustien aukkojen törmäyksistä, koska pimeät tähdet ovat isokokoisempia. On myös ehdotettu, että neutronitähdillä on pimeän aineen ydin, joka vaikuttaa siihen, millaisia gravitaatioaaltoja niiden törmäyksissä syntyy. Toistaiseksi on havaittu vain yksi törmäys, jossa ainakin toinen osapuoli on neutronitähti, mutta ensi vuoden alkupuolella toimintaan palaavan LIGO-havaintolaitteen ja sen seuralaisten odotetaan näkevän niitä tulevien vuosien kuluessa roppakaupalla. Suuri havaintomäärä auttaa pienten poikkeamien vaskaamisessa.

Ei tiedetä, koostuuko pimeä aine sekä hiukkasista että antihiukkasista, eikä sitä, ovatko niiden massat ja vuorovaikutukset sopivia pimeiden tähtien muodostamiseksi: voi olla, että niitä ei ole olemassa. Mutta idean tutkiminen osoittaa, miten fysiikan eri haaroilla –hiukkasfysiikalla, tähtien rakenteen tutkimisella ja yleisen suhteellisuusteorian gravitaatioaalloilla– voi olla yllättäviä yhtymäkohtia, ja että ongelmia kannattaa lähestyä monesta suunnasta.

21 kommenttia “Pimeä sydän”

  1. Eusa sanoo:

    Kylläpä sitä jaksetaan spekuloida villeillä hypoteeseilla… 🙂

  2. Juhani Harjunharja sanoo:

    Mielenkiintoista pohdintaa – ja samalla tavallaa runollisen mystistä: https://www.kirjavinkit.fi/arvostelut/pimea-sydan/

    Siis joka tapauksessa mielenkiintoista kuin Pimeä sydänkin.

  3. Erkki Kolehmainen sanoo:

    Tästä tulee mieleen Joseph Conradin romaani Pimeyden Sydän, johon myös Ilmestyskirja Nyt-elokuva löyhästi perustuu. Tuon fiktiivisen kertomuksen ja standardimallin pimeän aineen totuusarvo ovat samaa luokkaa.

  4. Leo Sell sanoo:

    Harrastelija

    Miten pimeän aineen tähdet ovat kirkkaita, eli mitä hiukkasia ne (oletetusti) säteilevät?

    1. Syksy Räsänen sanoo:

      Tekstissä lukee seuraavaa:

      ”Kun hiukkanen ja antihiukkanen kohtaavat, ne annihiloituvat kevyemmiksi hiukkasiksi, tyypillisesti enimmäkseen fotoneiksi ja neutriinoiksi.”

      Annihilaatiotuotteiden kirjo riippuu siitä, millaisesta pimeän aineen hiukkasesta tarkalleen on kysymys, mutta myös elektronit ja positronit ovat tavallisia annihilaatiotuotteita.

      Sanalla ”kirkkaus” tekstissä viittaan fotoneihin eli valoon.

  5. Miten tähden ulko-osien fysiikka mukaanlukien aurinkotuuli voisi riippua siitä tuottaako tähden ydin energiaa fuusiolla vai jollain muulla tavalla?

    1. Syksy Räsänen sanoo:

      Kun tähden pääasiallisena energianlähteenä on annihilaatio, sen lämpötila on pienempi kuin silloin kun päälähteenä on keskustan fuusioreaktiot (jotka vaativat korkean lämpötilan).

      Tarkemmin täällä: https://arxiv.org/abs/1501.02394

      1. Jännittävän kuuloinen ajatus heillä, ja erityisen mielenkiintoista jos voidaan verifioida lähitulevaisuudessa JWST:llä. Ehkä heidän mallinnuksensa on oikein, mutta olisin kyllä luullut päinvastoin että jos tähdelle lisätään jokin uusi energiantuottomekanismi, silloin tähdne kasvu hidastuisi eikä nopeutuisi – vaikkakaan DM ja fuusio eivät olisi käynnissä samanaikaisesti.

      2. Selitän epäilyäni vähän lisää. Karkeasti, ilman fuusiota musta aukko syntyisi tähdestä nopeasti. Fuusion kanssa se syntyy hitaammin. Kun mukaan pannaan DM-energiantuotanto, olettaisi että se syntyisi vieläkin hitaammin. Mutta heidän mukaansa käykin päinvastoin. Se on niin yllättävää että jäin pohtimaan olisiko heidän analyysissään jotain vialla.

        Ursan palvelin ei muuten nykyään näytä hyväksymistä odottavia kommentteja niinkuin se teki ennen, ainakaan minulle. Kun kommentin on lähettänyt, se katoaa näkyvistä kunnes ilmeisesti bloginpitäjä sen hyväksyy. Muutama viikko sitten tuo kiusa ilmaantui.

        1. Erkki Kolehmainen sanoo:

          Huomasin saman.

        2. Palle Juna sanoo:

          Voit ehkä ajatella niin että nynnyjen annihilaatiosta syntyvä säteily skalaautuu lempeämmin baryonisen massan funktiona. Tämä siis koska baryonit eivät sitä synnytä, vaan pimeä aine.

          Normaali tähti alkaa nopeasti säteillä todella kiihkeästi jos sinne kaataa lisää baryoneja, ja tämä säteily puskee tähden ulommat kerrokset kevyesti pois. Tavallisilla tähdillä onkin kohtalaisen matala yläraja massalle, wikipedian mukaan about 100 auringon massaa. Toisaalta tällaiset pimeät tähdet saattavat pysyä tasapainossa vaikka baryoninen massa olisi luokkaa miljoona auringon massaa.

          Näiden tähtien lopusta sanotaan seuraavaa: ”On the other end of the spectrum, the most massive supermassive dark stars may collapse directly to supermassive black holes with masses of >10^5 M⊙ without any fusion phase at all”

  6. Leo Sell sanoo:

    Harrastelija:

    ”Koska pimeä aine ei pysty jäähtymään”. Tarkoitatko että tavallisen aineen lämpötilan syntyessä hiukkastörmäyksistä -ei näitä törmäilyjä synny pimeän aineen kesken, jolloin ”lämpötilan” arvionti riippuu vain niiden kinetiikasta,muuttumattomista nopeuksista,joita verrataan valon nopeuteen?
    DM-anhilaatiotkin ovat törmäyksiä,joista lopulta tulee fotoneja – mutta ilmeisesti ilman lämpöhukkaa?

    1. Syksy Räsänen sanoo:

      Tavallinen aine jäähtyy siten, että muodostuu molekyylejä, joinen sisäinen tila voi muuttua niin, että ne säteilevät energiaa pois.

      Pimeä aine ei tiettävästi muodosta sidottuja tiloja (ainakaan tehokkaasti), joten se ei pysty säteilemään energiaa pois.

      1. Leo Sell sanoo:

        Harrastelija:kiitos vastauksestasi.Ihmettelen kuitenkin edelleen,miten pimeän aineen hiukkaset(wimpit=nynnyt) voivat muodostaa isoja rakenteita kuten tähtiä tai niiden osia,mikäli eivät voi tiivistyä pimeistä hiukkaspilvistä – aluksi ”viilenemällä” olkoonkin etteivät muodostaisi molaarisia sidostiloja?

        1. Syksy Räsänen sanoo:

          Pimeiden tähtien massasta vain alle tuhannesosa on pimeää ainetta, valtaosa on tavallista ainetta. Tähdet muodostuvat pimeän aineen pilvien keskelle, joten niiden sisällä on aina vähän pimeää ainetta.

          Pimeää ainetta tulee lisää tähtiin kahdella tapaa.

          Ensinnäkin ne vetävät sitä puoleensa (kuten muutakin ainetta) gravitaation takia.

          Toisekseen tähden läpi kulkevat pimeän aineen hiukkaset törmäilevät tavallisen aineen hiukkasten kanssa menettäen energiaa, jäävät siksi tähden vangiksi ja päätyvät lopulta keskustaan.

  7. Leo Sell sanoo:

    Harrastelija:Hieman lisäaineistoa luettuani, ymmärsin asian näin. Adiabaattinen kompressio supistaa kaasupilveä,joka on molaarisen vedyn (H2) viilentämä. DM-annihilaatioiden samalla voimistuessa,ne tuottavat lämpöenergiaa,mikäli näitä pimeitä hiukkasia on riittävän tiheässä ne viimein ionisoivat vedyn ympäriltään ja DS-eli Pimeä,mutta kirkas,tähti on syntynyt!

  8. Leo Sell sanoo:

    Harrastelija:

    Edellinen – tarkoitin:adiabaattinen kontraktio.

  9. Lentotaidoton sanoo:

    Eli suomeksi: sidottu tilahan tarkoittaa systeemin alinta energiatilaa (Enqvistin tunnettu sutkautus: sika painaa vähemmän kuin paloiteltu sika). Kun pimeä aine ei voi näitä muodostaa (kun eivät tunne sähkömagn vuorovaikusta) niin (ylimääräisen) energian säteileminen ei onnistu).

  10. Jussi Hakama sanoo:

    Hyvä Syksy

    Esittelemässäsi aiheessa oli paljon sellaista loogisuutta, mikä monesta muusta [yhtenäisteoria]teoretisoinnista taas tuntuu puuttuvan.

    Analogiaa geologiaan: kun tarkastelee teorioja, joita aikansa huippututkijat esittivät selittämään esim. jääkausista ja mannerliikunnoista seuranneita ilmiöitä, näkee, miten vaikea lause huippuälykkäällekin tutkijalle on ”En tiedä.” Kuitenkin se oli esim. Joseph Lagrangelle siinä määrin ominainen, että se taitaa olla kaiverrettuna hänen hautakivessäänkin.

    Käsi sydämelle: mikä ala-alue hallitsemallasi alueella tuntuu tässä suhteessa eniten ”haisevan”?

    Kunnioittavasti Jussi Hakama

    1. Syksy Räsänen sanoo:

      Aihe ei suoranaisesti liity mitenkään yhtenäisteorioihin.

      En ymmärrä kysymystä.

Vastaa

Sähköpostiosoitettasi ei julkaista. Pakolliset kentät on merkitty *